SlideShare uma empresa Scribd logo
1 de 56
Baixar para ler offline
pregunta:1
Complete el enunciado.
En un acto de comunicación, el es el conjunto de señales, signos o símbolos que son objeto de dicho
acto. El es el sistema de signos que se utiliza para dar a conocer información que circula por un conducto
físico llamado .
Opciones:
1
mensaje - código - canal
2
código - mensaje - canal
3
canal - código - mensaje
4
mensaje - canal - código
Correcta la opción:1
mensaje - código - canal
pregunta:2
Con base en el caso, relacione el elemento de la comunicación con el objeto que lo ejemplifica.
Un biólogo lee en su laboratorio una traducción al español del libro Fauna mundial, que trata sobre las ranas
marsupiales.
Opciones:
1
1a, 2b, 3c
2
1b, 2c, 3a
3
1c, 2d, 3b
4
1d, 2c, 3a
Elemento Objeto
1. Código a) Español
2. Canal b) Libro Fauna mundial
3. Contexto c) Laboratorio
d) Ranas marsupiales
Correcta la opción:1
1a, 2b, 3c
pregunta:3
Con base en el texto, relacione el elemento de la comunicación con su componente.
El adivino
Instalado en la plaza pública, un adivino se entregaba a su oficio. De repente se le acercó un vecino, anunciándole
que las puertas de su casa estaban abiertas y que habían robado todo lo que había en su interior.
El adivino levantose de un salto y salió corriendo hacia su casa, desencajado y suspirando, para ver lo que había
sucedido.
El vecino ledijo:
—Oye, amigo, tú que te vanaglorias de prever lo que les ocurrirá a los otros, ¿por qué no has previsto lo que te
sucedería a ti?
El adivino no supo qué responder […].
Modificado con fines pedagógicos. El adivino. Recuperado el 25 de enero de 2016 en http://bit.ly/1lLqXWN
Elemento Componente
1. Emisor a) Lenguaje común para el emisor y elreceptor
2. Receptor b) Las puertas de la casa estaban abiertas y se habían robado todo lo que había en su
interior
3. Mensaje c) El adivino
4. Código d) Unvecino
Opciones:
1
1a, 2c, 3d, 4b
2
1c, 2b, 3a, 4d
3
1d, 2a, 3b, 4c
4
1d, 2c, 3b, 4a
Correcta la opción:4
1d, 2c, 3b, 4a
pregunta:4
Con base en el caso, identifique el elemento de la comunicación que varía.
Stefhy White, docente de profesión e inglesa de nacimiento, ingresa al aula a paso firme. Haciendo gala de su
fuerte y firme voz, saluda en perfecto español a sus estudiantes, acto seguido apoya su bolso en el escritorio y
busca algo en su portafolio; encuentra una tiza blanca, se acerca a la pizarra y escribe: Por favor, silencio.
Opciones:
1
Emisor
2
Receptor
3
Código
4
Canal
Correcta la opción:4
Canal
pregunta:5
Identifique el elemento de la comunicación que varía en el caso.
Leticia era una mujer que le apasionaba hacer ejercicio semanalmente. Cierta ocasión contrajo una fuerte gripe
que le impidió realizar su acostumbrada rutina. Daniela, su vecina, le aconsejó que para curarse rápidamente se
tenía que hacer una infusión de leche con ajo y tomárselo en las mañanas. Luego, su madre le recomendó una
infusión de tilo. Finalmente, su instructor del gimnasio le sugirió tomarse una naranjada caliente con canela y miel
de abeja antes de dormir.
Opciones:
1
Receptor
2
Mensaje
3
Canal
4
Emisor
Correctalaopción:4
Emisor
pregunta:6
Relacione la función comunicativa con su característica.
Opciones:
1
1a, 2b, 3c
2
1b, 2a, 3c
3
1c, 2a, 3b
4
1c, 2b, 3a
Función Característica
1. Fática a) Influye en el comportamiento del interlocutor
2. Apelativa b) Toma como referencia al propio lenguaje
3. Metalingüística c) Inicia, prolonga, reanuda o finaliza la comunicación
Correcta la opción:3
1c, 2a, 3b
pregunta:7
Identifique la función del lenguaje en el texto.
Ahora que se ha comprobado que la harina de pescado es apta para el consumo humano, los científicos del
Gobierno están buscando recetas apetitosas para sacar provecho de sus cualidades de alto contenido de
proteínas. Uno de los candidatos es un líquido, la emulsión para bebés, elaborado a base de concentrado de
pescado, con el propósito de combatir la desnutrición en los infantes.
Camba, E. (2006).
Opciones:
1
Emotiva
2
Referencial
3
Poética
4
Metalingüística
Correcta la opción:2
Referencial
pregunta:8
Seleccione las funciones del lenguaje presentes en el texto.
El saxofonista Lester Young, fue el primero en decir “I am cool”, refiriéndose a que cool significa “estar relajado en un
determinado ambiente, con la situación bajo control" […]
Cool es una suerte de sublimación de aquello que jamás ha sido aceptado. Ponerle marca a lo no aceptado, a lo
freak, a lo mínimo, a lo “pasado de moda”. Ponerle un sello a la diferencia, y paradójicamente, normalizarla,
banalizarla. Quitarle su belleza original y venderla. Los empresarios sospechan que en la minoría hay una fuerza
latente peligrosa, y antes de que explote y de que sea una voz, la compran. Después la venden. Y nosotros la
compramos […]
Franco, A. Rebeldes que no se despeinan. Revista Mundo Diners. Quito. Recuperado el 18 de julio de 2014 en http://www.revistamundodiners.com/?p=4077
1. Apelativa
2. Emotiva
3. Informativa
4. Fática
5. Metalingüística
Opciones:
1
1, 2, 3
2
1, 3, 4
3
1, 3, 5
4
2, 4, 5
Correcta la opción:3
4
1, 3, 5
pregunta:9
Identifique la función del lenguaje.
[…] A todas horas de la noche, en las fiestas patrias, en el aniversario del descubrimiento de América, dispuesto a
solidarizarme con lo que sea, víctima de mi solidaridad. Inútil, completamente inútil, que me resista. La solidaridad
ya es un reflejo en mí, algo tan inconsciente como la dilatación de las pupilas. Si durante un centésimo de segundo
consigo desolidarizarme de mi solidaridad, en el centésimo de segundo que lo sucede, sufro un verdadero vértigo
de solidaridad. Solidario de las olas sin velas... sin esperanza. Solidario del naufragio de las señoras ballenatos, de
los tiburones vestidos de frac, que les devoran el vientre y la cartera. Solidario de las carteras, de los ballenatos y
de los fraques […].
Girondo, O. (s/f). Se podrá discutir mi erudición ornitológica y la eficacia de mis aperturas de ajedrez. Recuperado el 19 de noviembre de 2015 en http://bit.ly/1O6EiEM
Opciones:
1
Fática
2
Conativa
3
Estética
4
Metalingüística
Correcta la opción:3
Estética
pregunta:10
Con base en el texto, identifique el nivel de lenguaje.
La primera vez que se me ocurrió tentar la experiencia a cuyo relato están dedicadas estas líneas, fue una tarde,
leyendo no sé dónde, que los naturales de Java atribuían la falta de lenguaje articulado en los monos a la
abstención, no a la incapacidad. No hablan, decían, para que no los hagan trabajar.
Semejante idea, nada profunda al principio, acabó por preocuparme hasta convertirse en este postulado
antropológico:
Los monos fueron hombres que por una u otra razón dejaron de hablar. El hecho produjo la atrofia de sus órganos
de fonación y de los centros cerebrales del lenguaje; debilitó, casi hasta suprimirla, la relación entre unos y otros,
fijando el idioma de la especie en el grito inarticulado, y el humano primitivo descendió a ser animal.
Lugones, L. Izur. Recuperado el 10 de mayo de 2015 en http://www.ciudadseva.com/textos/cuentos/esp/lugones/yzur.htm
Opciones:
1
Coloquial
2
Culto
3
Vulgar
3
Familiar
Correcta la opción:2
Culto
pregunta:11
Identifique el nivel de lenguaje empleado en el texto.
Cierta noche, al regresar a casa, hallé, fijada a la puerta, una breve nota en que mi hija de 14 años de edad me
decía: “Adorada mamá, echa un vistazo al horno”. Allí encontré un segundo billete: “Está muy limpio, ¿no crees? Y
ahora, fíjate en el piso de la cocina”. Otras notas parecidas me fueron guiando por toda la casa, recién aseada y
puesta en orden. Sobre mi cama, un último papelito rezaba: “Te adoro, mamá”.
B. J. B. (1978). Selecciones del Reader’s Digest.
Opciones:
1
Culto
2
Vulgar
3
Coloquial
4
Científico
Correcta la opción:3
Coloquial
pregunta:12
Con base en el texto, seleccione los extranjerismos.
Iván era un músico talentoso que había perfeccionado su técnica como ninguno a la hora de interpretar la
balalaica. Su carisma lo había llevado a grandes escenarios alrededor del mundo y a recibir decenas de
distinciones como reconocimiento a su talento. Cuando era niño soñó con pelear en la guerra, pero una noche en
una reunión familiar, Iván vio que un jovencito que atendía el samovar desenfundó el curioso instrumento musical
y empezó a sacarle las más tristes melodías. El sonido impactó al niño y abandonó sus sueños bélicos para
dedicarse a revivir la pasión de su pueblo por el olvidado instrumento.
1. Balalaica
2. Escenarios
3. Distinciones
4. Guerra
5. Samovar
6. Bélicos
Opciones:
1
1, 2, 6
2
1, 4, 5
2, 3, 4
4
3, 5, 6
Correctalaopción:2
1, 4, 5
pregunta:13
Relacione la oración con el nivel de lenguaje utilizado.
Nivel Oración
Opciones:
1
1b, 2a, 3c, 4d
2
1b, 2c, 3a, 4d
3
1d, 2c, 3a, 4b
4
1d, 2c, 3b, 4a
1. Vulgar a) Este es un hábitat de fauna endémica
2. Formal b) Mi hijo trajo a sus guaguas
3. Científico c) Tenga la gentileza de sentarse
4.
Familiar d)
Es la misma señora que vino
denantes
Correcta la opción:3
1d, 2c, 3a, 4b
pregunta:14
En un periódico local se han impreso varios titulares, identifique el que usa extranjerismos.
Opciones:
1
Triunfan los llapingachos en festival de comida internacional 2
Cambio en hipertexto afectó a los mensajes en las principales redes 3
Gobierno entregará ordenadores a estudiantes de comunidades rurales 4
Vecinos denuncian un parking improvisado junto a un terreno
Correcta la opción:4
Vecinos denuncian un parking improvisado junto a un terreno
pregunta:15
Seleccione los enunciados que contienen extranjerismos.
1. Las palabras pronunciadas por el autor parecían adecuadas al cliché
2. En el mundo del deporte, aquel joven tenista es considerado como un monstruo
3. Me encanta cenar en este restaurante porque su cocinero es todo un gourmet
4. Los actores llegaron temprano para ensayar los diálogos y cumplir surol
5. Tenía muy irritados los ojos y el exceso de polución le causabaprurito
6. Después de aprobar todos los cursos, empezarán el periodo de praxis
Opciones:
1
1, 2, 5
2
1, 3, 4
3
2, 4, 6
4
3, 5, 6
Correctalaopción:2
1, 3, 4
pregunta:16
Elija las palabras que se forman con prefijos.
1. Medidor
2. Primavera
3. Inconveniente
4. Inmortalizar
5. Kilómetros
6. Penúltimo
Opciones:
1
1, 2, 3, 4
2
1, 2, 4, 5
3
2, 3, 5, 6
4
3, 4, 5, 6
Correctalaopción:4
3, 4, 5, 6
pregunta:17
Seleccione las palabras formadas con prefijos.
1. Destellante
2. Franquicia
3. Desmesura
4. Tetracordio
5. Posadeño
6. Posponer
Opciones:
1
1, 2, 5
2
1, 3, 6
3
2, 4, 5
4
3, 4, 6
Correctalaopción:4
3, 4, 6
pregunta:18
Seleccione las palabras formadas con prefijos.
1. Abadía
2. Anormalidad
3. Acuático
4. Adyacente
5. Antropomorfo
6. Anacrónico
Opciones:
1
1, 3, 5
2
1, 4, 6
3
2, 3, 5
4
2, 4, 6
Correctalaopción:4
2, 4, 6
pregunta:19
Seleccione las palabras formadas con sufijos.
1. Brillantez
2. Producto
3. Sacudida
4. Egoísmo
5. Policéfalo
6. Torno
Opciones:
1
1, 2, 6
2
1, 3, 4
3
2, 4, 5
4
3, 5, 6
Correctalaopción:2
1, 3, 4
pregunta:20
Seleccione las palabras formadas por el prefijo que signifca negación.
1. Descarga
2. Amorfo
3. Incógnito
4. Impecable
5. Anarquista
6. Sinfonía
Opciones:
1
1, 2, 3, 4
2
1, 3, 5, 6
3
2, 3, 4, 5
4
2, 4, 5, 6
Correctalaopción:3
2, 3, 4, 5
pregunta:21
Seleccione las palabras que se componen con sufijos despectivos.
1. Caserío
2. Aldehuela
3. Viejísima
4. Libraco
5. Policastro
6. Peineta
Opciones:
1
1, 3, 6
2
2, 3, 5
3
2, 4, 5
4
3, 4, 6
Correctalaopción:3
2, 4, 5
pregunta:22
Con base en el texto, identifique la palabra formada con el prefijo que significa privación.
En sus páginas interminables hay mapas de lugares trastocados por la memoria, y planos de ciudades que se
han convertido en silencio; los recuerdos de un hombre que caminó toda la vida alrededor de su cama hasta sentir
que descubrió el Polo Norte y la larga canción sin música de un flautista demencial que jura poder hablar con las
cigüeñas.
Modificado con fines pedagógicos. Recuperado el 3 de marzo de 2017 en http://bit.ly/2mUXTO3
Opciones:
1
Trastocados
2
Interminables
3
Flautista
4
Recuerdos
Correcta la opción:2
Interminables
pregunta:23
Identifique la palabra compuesta por el prefijo que significa dificultad o anomalía.
Opciones:
1
Sinfonía
2
Perifonía
3
Disfonía
4
Afonía
Correcta la opción:3
Disfonía pregunta:24
Identifique la palabra compuesta por el prefijo que significa a la vez.
Opciones:
1
Subconjunto 2
Analgesia
3
Sincronía
4
Adverbio
Correcta la opción:3
Sincronía
pregunta:25
Identifique el tiempo en el que se encuentra conjugado el verbo en la oración.
Todas las tardes venía por el pan.
Opciones:
1
Pretérito perfecto compuesto 2
Pretérito perfectosimple 3
Pretérito pluscuamperfecto 4
Pretérito imperfecto
Correcta la opción:4
Pretérito imperfecto
pregunta:26
Identifique el tiempo del verbo resaltado en negrita.
Él leía novelas cortas para entretenerse.
Opciones:
1
Presentesubjuntivo
2
Pretéritoimperfecto 3
Pretéritoindefinido 4
Potencial simple
Correcta la opción:2
Pretérito imperfecto
pregunta:27
Identifique la forma verbal conjugada en pretérito perfecto.
La víspera, en la catedral se celebraba una solemne ceremonia fúnebre que impidió que se hicieran preparativos
de ninguna clase. Como consecuencia, en una sola mañana había que vestir con colgaduras de damasco rojo
todas las columnas góticas de las tres naves, que tenían unos treinta pies de elevación. El obispo había mandado
venir de París cuatro tapiceros; pero esos señores no lograban hacerlo todo, y por añadidura, tampoco tenían
grandes ganas de trabajar.
Modificado con fines pedagógicos. Stendhal. (s/f). Rojo y negro. Recuperado el 19 de noviembre de 2015 en http://bit.ly/SZqXVp
Opciones:
1
Celebraba
2
Tenían
3
Impidió
4
Lograban
Correcta la opción:3
Impidió
pregunta:28
Seleccione las partes variables de la oración.
La casa de mi abuelo tiene un jardín muy amplio.
1. La
2. De
3. Abuelo
4. Un
5. Muy
Opciones:
1
1, 2, 3
2
1, 3, 4
3
2, 3, 5
4
2, 4, 5
Correctalaopción:2
1, 3, 4
pregunta:29
Identifique las partes invariables de la oración.
En el zoológico hay algunos monos, osos, tortugas, llamas, tigrillos y un cóndor.
Opciones:
1
zoológico - un
2
el - hay
3
algunos - cóndor
4
en - y
Correcta la opción:4
en - y
pregunta:30
Con base en el texto, elija las palabras variables.
Lo que aquel hombre asimiló fue que el capullo restrictivo y la lucha eran necesarias para que la mariposa
pudiera salir por el diminuto agujero, era la manera que utilizaba la naturaleza para enviar fluido del cuerpo de la
mariposa hacia sus alas, de modo que estuviera lista para volar tan pronto obtuviere la libertad del capullo. Todo
esfuerzo es vital para capturar nuestras metas.
Modificado con fines pedagógicos. Recuperado el 06 de septiembre de 2017 en http://www.doslourdes.net/la_lecci%C3%B3n_de_la_mariposa.htm
1. Que
2. Restrictivo
3. De modo que
4. Asimiló
5. Para que
6. Obtuviere
Opciones:
1
1, 4, 5
2
1, 5, 6
3
2, 3, 5
4
2, 4, 6
Correctalaopción:4
2, 4, 6
pregunta:31
Identifique una parte invariable de la oración.
Mas no pasó mucho tiempo para que el águila recibiera el pago de su traición.
Opciones:
1
Mas
2
Pasó
3
Su
4
El
Correcta la opción:1
Mas
pregunta:32
Complete el enunciado.
En la oración "El perro anciano tiene ahora una expresión de ensueño"; las palabras y son
invariables porque carecen de inflexiones.
Opciones:
1
perro -anciano
2
el - una
3
expresión -ensueño
4
ahora - de
Correcta la opción:4
ahora - de
pregunta:33
Elija las partes variables de la oración.
Luis o Karla nos contarán todo cuando regresen.
1. Luis
2. O
3. Nos
4. Contarán
5. Cuando
6. Regresen
Opciones:
1
1, 2, 3, 5
2
1, 3, 4, 6
3
2, 4, 5, 6
4
3, 4, 5, 6
Correctalaopción:2
1, 3, 4, 6
pregunta:34
Seleccione las partes variables de la oración.
La poesía es un territorio que salva de la intemperie.
1. Es
2. Un
3. Que
4. De
5. La
Opciones:
1
1, 2, 5
2
1, 3, 4
3
2, 3, 5
4
3, 4, 5
Correctalaopción:1
1, 2, 5
pregunta:35
Identifique las conjunciones.
1. Sino
2. En
3. Y
4. Que
5. Hasta
6. Un
7. Pero
Opciones:
1
1, 2, 5, 7
2
1, 3, 4, 7
3
2, 3, 4, 6
4
3, 5, 6, 7
Correctalaopción:2
1, 3, 4, 7
pregunta:36
Identifique la función sintáctica del grupo nominal en negrita.
El vocablo yo-yo o yoyó, pues de las dos formas se puede escribir, procede del tagalo, la lengua nativa de
Filipinas, y significa volver. Hasta hace 400 años, los filipinos usaban el yoyó como arma. No obstante, el origen de
este juguete parece estar en China, aunque también era conocido en Grecia por lo menos hace 2 500 años.
Los artesanos helenos lo fabricaban con madera, metal o terracota, y decoraban cada pieza con dibujos de los
dioses. Algunos de estos primeros yoyós pueden contemplarse en el Museo Metropolitano de Arte, en Nueva York.
Muy Interesante. ¿Quién inventó el yoyó? Recuperado el 27 de enero de 2016 en bit.ly/23t9Qen
Opciones:
1
Objeto indirecto
2
Complemento circunstancial 3
Objeto directo
4
Núcleo del sintagma verbal
Correcta la opción:2
Complemento circunstancial
pregunta:37
Identifique la función sintáctica del sintagma en negrita.
Se convirtieron en mi mayor tesoro los viejos libros de la biblioteca.
Opciones:
1
Aposición
2
Núcleo del sujeto
3
Modificadordirecto 4
Modificador indirecto
Correcta la opción:4
Modificador indirecto
pregunta:38
Relacione el elemento sintáctico con su definición.
Elemento Definición
1.
Complemento
indirecto a)
Determinante del núcleo del predicado que señala alguna circunstancia particular en
que ocurre la acción del verbo
2. Núcleo del sujeto b)
Construcción sintáctica formada por un término unido al núcleo del sujeto por
medio de un nexo, su función es explicativa
3.
Complemento
circunstancial
Opciones:
1
c) Sustantivo o pronombre que realiza la acción del predicado
1a, 2b, 3c
2
1b, 2c, 3a
3
1c, 2a, 3b
4
1c, 2b, 3a
Correcta la opción:2
1b, 2c, 3a
pregunta:39
Identifique la definición del objeto indirecto.
Opciones:
1
Parte de la oración que recibe de forma directa la acción del verbo y se reconoce porque se sustituye con los
pronombres personales átonos "lo", "la", "los" o "las"
2
Elemento sintáctico que acompaña al núcleo del sujeto de manera directa en forma de artículo o adjetivo y
concuerda con él en género y número
3
Parte de la oración compuesta por un conjunto de palabras que se añaden al núcleo del sujeto a través de
preposiciones
4
Elemento sintáctico que evidencia al receptor de los efectos de la acción verbal, va precedido por una de las
preposiciones "a" o "para", y puede ser reemplazado por los pronombres "le" o "les"
Correcta la opción:4
Elemento sintáctico que evidencia al receptor de los efectos de la acción verbal, va precedido por una
de las preposiciones "a" o "para", y puede ser reemplazado por los pronombres "le" o "les"
pregunta:40
Con base en el texto, identifique la función sintáctica de los términos en negrita.
El vocablo yo-yo o yoyó, pues de las dos formas se puede escribir, procede del tagalo, la lengua nativa de
Filipinas, y significa volver. Hasta hace 400 años, los filipinos usaban el yoyó como arma. No obstante, el origen de
este juguete parece estar en China, aunque también era conocido en Grecia por lo menos hace 2 500 años. Los
artesanos helenos lo fabricaban con madera, metal o terracota y decoraban cada pieza con dibujos de los dioses.
Algunos de estos primeros yoyós pueden contemplarse en el Museo Metropolitano de Arte, enNueva York.
Muy Interesante. ¿Quién inventó el yoyó? Recuperado el 27 de enero de 2016 en bit.ly/23t9Qen
Opciones:
1
Modificadordirecto 2
Aposición
3
Modificador indirecto
4
Núcleo del sintagma nominal
Correcta la opción:3
Modificador indirecto
pregunta:41
Identifique la función del sintagma en negrita.
El episodio final de la serie, cual primicia noticiosa, será visto por millones de espectadores.
Opciones:
1
Predicativo
2
Agente
3
Complemento circunstancial 4
Complemento indirecto
Correcta la opción:2
Agente
pregunta:42
Identifique el objeto directo en la oración.
Las docentes de Guayaquil escriben poesías para sus estudiantes.
Opciones:
1
Escriben
2
Docentes
3
Poesías
4
Guayaquil
Correcta la opción:3
Poesías
pregunta:43
Identifique la función del sintagma en negrita.
La joven doctora, Mariana Acosta, es una investigadora mundialmente conocida por sus descubrimientos
genéticos.
Opciones:
1
Complemento directo 2
Complementoindirecto 3
Agente
4
Atributo
Correctalaopción:4
Atributo
pregunta:44
Relacione el vicio de dicción con la oración que lo ejemplifica.
Opciones:
1
1a, 2c, 3e, 4b,
5d 2
1b, 2e, 3d, 4a,
5c 3
1c, 2e, 3a, 4d,
5b 4
1d, 2a, 3c, 4e,
5b
Vicio Oración
1. Cacofonía a) Salga afuera a ver si llegó el estudiante
2. Pleonasmo b) Clickea en tu mouse a ver si funciona
3. Muletilla c) Este, dime de nuevo cómo pasó
4. Solecismo d) Paúl, para para pensar
5. Neologismo e) Bajo el punto de vista del señor, se podría decir que es así
Correcta la opción:4
1d, 2a, 3c, 4e, 5b
pregunta:45
Seleccione las oraciones en las que se ha incurrido en dequeísmo.
1. Me enteré de que llegarías después de las vacaciones de invierno
2. Los últimos en llegar nos dijeron de que todo estaba muy bien
3. Los empleadores creyeron que la cuenta estaba cerrada desde ayer
4. Insistieron de que tratáramos de venir pronto
Opciones:
1
1, 3
2
1, 4
3
2, 3
4
2, 4
2, 4
pregunta:46
Relacione el vicio de dicción con su ejemplo.
Vicio Ejemplo
1. Cacofonía a) Me enteré que esa relación de años se ha rompido por nimiedades
2. Conjugación verbal incorrecta b) La agua oxigenada tiene propiedades cicatrizantes
3. Neologismo c) Debes ser proactiva en tutrabajo
d) Sé que en ese cubículo cabo sin problema
e) Yo lo coloco y ella lo quita
f) Solo debes clickear en el enlace de la red
Opciones:
1
1ab, 2df, 3ce
2
1bc, 2ad, 3ef
3
1be, 2ad, 3cf
4
1ef, 2bc, 3ad
Correcta la opción:3
1be, 2ad, 3cf
pregunta:47
Relacione el vicio de construcción con la oración que lo ejemplifica.
Vicio Oración
1. Anfibología a) La situación se puso intransigente
2. Dequeísmo b) Contó que cuando ya no aguantó más, salió afuera a llorar
3. Redundancia c) Le dijo de que volviera mañana en la tarde
4. Uso incorrecto de acuerdo con el
significado
d) No puedo accesar al programa para printear el
documento
5. Barbarismo e) Salió tan impresionada de la película que temblaba
Opciones:
1
1b, 2c, 3e, 4d,
5a 2
1c, 2b, 3d, 4e,
5a 3
1d, 2e, 3a, 4b,
5c 4
1e, 2c, 3b, 4a, 5d
1e, 2c, 3b, 4a, 5d
pregunta:48
Identifique la oración que presenta un vicio de construcción.
Opciones:
1
Debes de obedecer a tu papá
2
Asistimos al concierto de salsa
3
Trabajo tanto que me siento estresada 4
Los estudiantes deben desarrollar habilidades lingüísticas
Correcta la opción:1
Debes de obedecer a tu papá
pregunta:49
Identifique el enunciado que presenta un vicio de palabra o cláusula a la que puede darse más de una
interpretación.
Opciones:
1
Le llamó para confirmar que sería contratado 2
Estamos convencidos que ganaremos el juego 3
Me informó de que no era necesario reportarlos 4
Se arrienda habitación para estudiantes con cama
Correcta la opción:4
Se arrienda habitación para estudiantes con cama
pregunta:50
Identifique el enunciado que presenta un caso de queísmo.
Opciones:
1
¡Qué emoción! El próximo fin de semana regresan mis familiares 2
Me avisaron de que mañana saldremos a caminar por la carretera 3
Nunca cayó en cuenta que había olvidado las llaves de su casa antes de salir 4
Todavía no entiendo sus decisiones, ¿qué razones aparentaban lo contrario?
Correcta la opción:3
Nunca cayó en cuenta que había olvidado las llaves de su casa antes de salir
pregunta:51
1, 3, 5
Identifique la oración en la que la coma cumple la función de limitar una aclaración o ampliación.
Opciones:
1
José tenía como juguetes un balón, un carro, una patineta y una bicicleta 2
Los jugadores saltaron al campo de juego, el público enloquecía y las barras no dejaban de alentar 3
Descartes, notable matemático francés, hizo muchos aportes en sus escritos 4
Juan, ven por favor
Correcta la opción:3
Descartes, notable matemático francés, hizo muchos aportes en sus escritos
pregunta:52
Seleccione las oraciones que utilizan de forma correcta los signos de puntuación.
1. La bandera ecuatoriana es amarilla azul y roja.
2. Guayaquil, puerto principal del Ecuador, es famosa por su gastronomía.
3. Mi hijo mayor practica baloncesto, el segundo, fútbol, y el tercero, tenis.
4. Tania no preparó los exámenes; sin embargo, pasó al siguiente curso.
5. Tomás no paraba de llorar; por consiguiente, fue hospitalizado.
Opciones:
1
1, 2, 4
2
1, 3, 5
3
2, 3, 4
4
2, 4, 5
Correctalaopción:4
2, 4, 5
pregunta:53
Seleccione las oraciones que utilizan correctamente las normas de puntuación.
1. ¿Se le ofrece café, té, o agua?
2. La región Amazónica es extensa, exuberante y calurosa.
3. Las puertas, ventanas, y armarios quedaron destruídos.
4. Él, que no era tonto, se dio cuenta al instante.
5. Cortaron los árboles, limpiaron el jardín, y quedaron exhaustos.
Opciones:
1
1, 2, 4
2
2, 4, 5
3
2, 3, 5
4
2, 4, 5
Correctalaopción:4
2, 4, 5
pregunta:54
Seleccione las oraciones en las que falta colocar punto y coma (;).
1. El bebé tomó su cena, jugó un rato más y se quedó dormido en brazos de su madre.
2. Mi madre trajo dulces, empanadas y refrescos en seguida se puso a preparar la comida.
3. Llovió toda la noche. Al otro día los campos amanecieron inundados y empantanados.
4. Los síntomas de la gripe empeoraron: tos, catarro, fiebre para colmo me dolía la cabeza.
5. Parecía muy contento con el trabajo, la compañía y todo pero al otro mes estaba agotado.
Opciones:
1
1, 2, 4
2
1, 3, 4
3
2, 3, 5
4
2, 4, 5
Correctalaopción:4
2, 4, 5
pregunta:55
Elija los enunciados en los que se utilizan correctamente las normas de acentuación.
1. La niña que esta frente a Juan es una de las más inteligentes del salón
2. ¡Esta no es vida! Gritó el pescador y se echó a la mar mientras silbaba la tonada
3. No era necesaría tanta explicación para entender el concepto galo detruhan
4. Ayer, todo estuvo listo para la grabación: las luces, el guion, las cámaras, el set…
5. Cuando me habló de estudiar Astronomía, creí que era una de sus tantasbromas
Opciones:
1
1, 2, 4
2
1, 3, 5
3
2, 3, 4
4
2, 4, 5
Correcta la opción:4
pregunta:56
Elija las oraciones que respetan las reglas de acentuación.
1. El perro de Juan siempre juega con él
2. María se fue a trabajar, eso yo lo se
3. Quería hacer más ejercicio, mas tuve que volver a mi casa
4. Mi auto está pintado de un color pedido especialmente por mí
5. Si yo dijera siempre que si, estaría perdido
6. De vez en cuando, las personas necesitan que se les dé un consejo
Opciones:
1
1, 2, 3, 5
2
1, 3, 4, 6
3
2, 3, 4, 5
4
2, 4, 5, 6
Correctalaopción:2
1, 3, 4, 6
pregunta:57
Elija las oraciones en las que se emplean de forma adecuada las mayúsculas.
1. Me dijeron que María está haciendo compras en el mercado con luisa
2. Juan y Pedro son maestros de lengua y literatura
3. La federación de fútbol de Brasil inaugura el mundial en junio
4. Los estudiantes de séptimo año de educación básica fueron de paseo aLoja
5. El hospital doce de octubre es el mejor de la ciudad
6. El Viernes Santo es un día de descanso obligatorio para los empleadospúblicos
Opciones:
1
1, 3, 4
2
2, 3, 5
3
2, 4, 6
4
4, 5, 6
Correctalaopción:3
2, 4, 6
pregunta:58
Identifique la oración en la que se usan correctamente las mayúsculas.
Opciones:
1
La brújula señala el Norte
2
El Norte de la ciudad está inundado 3
Viajamos por el Norte de Ecuador
4
El viento del Norte es más suave
Correcta la opción:1
La brújula señala el Norte
pregunta:59
Identifique el enunciado que contiene el conector que expresa contraste.
Opciones:
1
Sus amigos guardaron silencio, a la vez, su familia aplaudió complacida 2
Sus amigos guardaron silencio, asimismo, su familia aplaudió complacida 3
Sus amigos guardaron silencio, en cambio, su familia aplaudió complacida 4
Sus amigos guardaron silencio, por último, su familia aplaudió complacida
Correcta la opción:3
Sus amigos guardaron silencio, en cambio, su familia aplaudió complacida
pregunta:60
Identifique el tipo de conector que se utiliza en la oración.
Después de la comida, tomaron un café en el salón.
Opciones:
1
Causal
2
Adversativo
3
Comparativo
4
Temporal
Correcta la opción:4
Temporal
pregunta:61
Relacione la palabra con la oración en la que se encuentra su antónimo.
Palabra Oración
1. Tácito a) El gato de la casa es muy arisco
2. Amigable b) Aquel hombre la trataba con descortesía
3. Amabilidad c) Lo que dice no está expreso
Opciones:
1
1a, 2c, 3b
2
1b, 2a, 3c
3
1c, 2a, 3b
4
1c, 2b, 3a
Correcta la opción:3
1c, 2a, 3b
pregunta:62
Identifique el antónimo de la palabra en negrita.
[…] Con todos aquellos personajes de la aristocracia parisina fue tejiendo otro mundo literario en una evocación
magistral que, más que copiarla según el realismo tradicional, lo evoca con la sutileza de un pintor impresionista.
De hecho, la novela de Proust se ha comparado con la pintura de Monet y la música de Debussy por su pretensión
de captar la luz y el instante, y de extraer de la realidad la impresión fugaz de la misma […].
Ponte Far, J. (2015). Marcel Proust, uno de los grandes. Recuperado el 16 de noviembre de 2015 en http://bit.ly/1RZHpxw
Opciones:
1
Duradera
2
Efímera
3
Transitoria
4
Perecedera
Correcta la opción:1
Duradera
pregunta:63
Seleccione las oraciones que contienen palabras homógrafas.
1. ¡Que se alce el sol y brille! -dijo el granjero aquella mañana. En el prado divisó un alce que se alimentaba del
pasto
2. Ese instante, una hermosa mujer abría la puerta y se acercaba hacia él... habría de ser un día feliz
3. Su esposa había regresado, podrían seguir juntos su camino. "Hoy camino contigo" le dijo ella
4. A pesar de coser la ropa y de cocer la fruta para las mermeladas, la mujer mantenía su buen humor
5. Gracias a ella, la llama de su relación seguía. Su nombre reflejaba su carácter, "se llama Alegría"
Opciones:
1
1, 2, 5
2
1, 3, 4
3
1, 3, 5
4
2, 3, 4
Correctalaopción:3
1, 3, 5
pregunta:64
Seleccione las palabras homógrafas.
1. Acerbo
2. Prensa
3. Alce
4. Vacilo
5. Vino
Opciones:
1
1, 2, 3
2
1, 3, 4
3
2, 3, 5
4
2, 4, 5
Correctalaopción:3
2, 3, 5
pregunta:65
Seleccione las características propias de una composición satírica.
1. Se manifiesta en tono mordaz
2. Presenta temas con los que un grupo humano se identifica
3. Exterioriza los lamentos tras la defunción de un ser querido
4. Expone al ridículo a alguien o algo
Opciones:
1
1, 3
2
1, 4
3
2, 3
4
2, 4
Correcta la opción:2
1, 4
pregunta:66
Con base en el texto, identifique el subgénero literario.
A un niño de un pueblo le regalaron un pequeño pájaro. Al recibirlo, el niño le ató una pata a un hilo y lo mantuvo
agarrado. Luego lo lanzó para que volara, pero el animal se quedó quieto. El niño le dijo: - Animal lento, te doy
permiso y espacio para que extiendas tus alas y vueles, pero tú te quedas totalmente quieto. No sabes gozar de tu
libertad. El pájaro, molesto, le contesto: - No soy lento, solo soy astuto. Esa libertad que me das es solo apariencia
y ese bien pronto se tornará en un mal. Apenas intente volar, tú me jalarás por el hilo atado a mi pata.
“No hay peor esclavitud que una en la que se crea que se es libre, cuando no lo es”.
El niño y el pájaro. (s.f.). Recuperado el 07 de abril de 2017 en http://bit.ly/2oMJFTT
Opciones:
1
Cuento
2
Fábula
3
Leyenda
4
Mito
Correcta la opción:2
Fábula
pregunta:67
Eleonor debe elaborar un ensayo que justifique que Otelo es una tragedia, por lo que, con el fin de evidenciar características de
este subgénero, ha realizado una cita textual de la obra. Con base en el fragmento, identifique el argumento que eligió Eleonor
para desarrollar su tarea.
EMILIA.- ¡Buenos caballeros, permitidme que hable! Es justo que lo obedezca, pero no ahora. ¡Quizá, Yago, no vuelva nunca al
hogar!
OTELO.- ¡Oh, oh, oh! (Cae sobre el lecho)
EMILIA.-¡Sí! ¡Déjate caer y ruge! ¡Pues ha matado a la más tierna inocente que alzó jamás los ojos al cielo!
OTELO.- (Levantándose) ¡Oh! ¡Era impura! (A Graciano) Apenas os conozco, tío. ¡Ahí yace vuestra sobrina, cuyo aliento, en
verdad, acaban de cortar estas manos! ¡Sé que este acto aparece horrible y cruel!
GRACIANO.- ¡Pobre Desdémona! ¡Cuánto me alegro de que no exista tu padre! ¡Tu casamiento fue para él un golpe mortal, y
la sola pena que cortó en dos el viejo hilo de su vida! Si viviera ahora, este espectáculo le impulsaría a algún acto de
desesperación. ¡Sí! ¡Maldeciría a su buen ángel, le arrojaría de su lado y se atraería la reprobación del cielo!
Shakespeare, W. (s.f.). Otelo: el moro de Venecia. Recuperado el 05 de abril de 2017 en http://bit.ly/1DQCtaV
Opciones:
1
Este fragmento denota un ámbito de conflicto en el que se aprecian realidades normales de la vida cotidiana, por lo
tanto, corresponde a una tragedia
2
Este fragmento muestra la interacción de varios personajes: Emilia, Otelo y Graciano, de forma chabacana y
grotesca, por lo tanto, corresponde al subgénero de la tragedia
3
En la obra Otelo: moro de Venecia, la acción dramática desemboca en un mensaje religioso, basado en temas
morales o sacros, por ello es una tragedia
4
En la obra Otelo: moro de Venecia, las escenas reflejan situaciones centradas en el sufrimiento y las peripecias
dolorosas de la vida humana, por lo tanto, es una tragedia
Correcta la opción:4
En la obra Otelo: moro de Venecia, las escenas reflejan situaciones centradas en el sufrimiento y las
peripecias dolorosas de la vida humana, por lo tanto, es una tragedia
pregunta:68
Con base en el fragmento, identifique el tipo de texto no literario.
- Ha escrito usted un libro muy divertido.
- Quise hacerlo así. La verdad es que publicar libros es divertido. Para mí hubiera sido muy difícil escribir sobre el
mundo editorial sin sentido del humor.
- ¿Siempre ha sido así?
- Siempre. Cada libro, cada manuscrito que entra por esa puerta es lo más importante en la vida del autor. Hay
excepciones entre los libros que se escriben y que se publican, pues hay libros para perder peso, diccionarios,
libros para hacer mejor el amor; pero los que me llegan a mí representan, sin excepciones, lo más importante en la
vida de sus autores […]
Cruz, J. (6 de noviembre de 2005). El editor en el trapecio. El País (archivo digital). Recuperado el 24 de julio de 2014 en http://bit.ly/1rOVAJC
Opciones:
1
Noticia
2
Entrevista
3
Editorial
4
Crónica
Correcta la opción:2
Entrevista
pregunta:69
Identifique el tipo de texto.
He aquí uno de los géneros más apasionados e ingratos del periodismo: la entrevista a una estrella de rock. La
principal constatación de quienes hemos pasado por el calvario de aguardar a una luminaria en el banquillo de las
expectativas es que ningún héroe del alto volumen quiere hablar. Profesionales de la intensidad, quienes expresan
adrenalina en un escenario suelen relajarse con otras formas del exceso, de la destrucción de televisores al acopio
de pandas de peluches, motocicletas, castillos medievales y otras variantes del coleccionismo extremo. La
entrevista es una convención tediosa.
Villoro, J. (2016). La entrevista a una estrella de rock. Modificado con fines pedagógicos. Recuperado el 18 de febrero de 2016 en http://bit.ly/1PIxkVJ
Opciones:
1
Crónica
2
Artículo deopinión
3
Noticia
4
Reportaje científico
Correcta la opción:2
Artículo de opinión
pregunta:70
Identifique la clase de texto no literario que se muestra a continuación.
Cuatro nuevas especies de ranas terrestres fueron halladas en los bosques del río Upano, provincia de Morona
Santiago, en la vertiente oriental de la cordillera de los Andes. Esto, tras un estudio realizado por tres biólogos
ecuatorianos que evaluaron ocho localidades de la subcuenca alta del río. Las expediciones les permitieron
obtener abundante información sobre las ranas terrestres Pristimantis, con el objetivo de construir una sinopsis del
género, identificar especies nuevas y determinar sus patrones de diversidad.
El estudio revela que la cuenca del río Upano contiene 37 especies de ranas terrestres. De estas, 25 han sido
previamente reportadas en los bosques de los Andes orientales de Ecuador, 16 de ellas son endémicas, nueve
amenazadas, ocho son descritas como especies candidatas a nuevas y cuatro son identificadas en el estudio como
nuevas.
Alarcón, I. (2017). Ecuador cuenta con cuatro nuevas especies de ranas terrestres. Recuperado el 4 de abril de 2017 en http://bit.ly/2nPN2FV
Opciones:
1
Editorial
2
Crónica
3
Noticia
4
Reseña
Correcta la opción:3
Noticia
pregunta:71
Identifique el tipo de texto no literario.
Desde hace un tiempo, viene dándose un fenómeno curioso en los segmentos de ciencia que se presentan en
varios noticieros televisivos. Cada vez que se hace un alegato en defensa de la ciencia, o se presenta a científicos
que reclaman más ayudas, presupuestos o becas, se muestra en pantalla a gente en bata. Personalmente, me
preocupa que se genere la sensación de que la única ciencia necesaria es aquella que tiene un impacto inmediato
en la sociedad. No pretendo que se reduzca la visibilidad de estos heroicos
investigadores, lo único que pretendo es recordar que la investigación científica es necesaria en todos los ámbitos,
desde los que tienen aplicaciones tangibles hasta los más teóricos y abstractos. Es peligroso abandonar toda esa
“otra” ciencia por el mero hecho de que venda menos.
Modificado con fines pedagógicos. Moral, D. (2017). Ciencia hay más de una. Recuperado el 18 de abril del 2017 en http://bit.ly/2oIPdxY
Opciones:
1
Reportajecientífico
2
Noticia
3
Artículo deopinión
4
Crónica
Correcta la opción:3
Artículo de opinión
pregunta:72
Identifique la idea principal en el fragmento.
Los apodos y sobrenombres existen porque todos los seres humanos nos prestamos para ser mencionados de tal o
cual manera, porque algo, aunque sea pequeñito, nos hace únicos y diferentes, y porque estamos expuestos a la
lupa de un sinnúmero de observadores. Uno de los mejores caldos de cultivo para los alias es el colegio, pero no es
el único, no podemos dejar de lado a los personajes de la política, periodistas, vecinos, deportistas y familiares que
seguramente serán motivo de inspiración para un nuevo artículo.
León, R. (2008). De apodos, motes y sobrenombres. Revista Mundo Diners. Nro. 318. Quito. p. 93.
Opciones:
1
En cuestión de apodos no podemos dejar de lado a políticos, periodistas, vecinos, deportistas y familiares que
inspirarán nuevos alias
2
Todos los seres humanos, sin excepción, tenemos sobrenombres o apodos 3
Los sobrenombres existen porque somos únicos y estamos expuestos a un sinnúmero de observadores 4
Uno de los mejores caldos de cultivo para los sobrenombres es el colegio
Correcta la opción:3
Los sobrenombres existen porque somos únicos y estamos expuestos a un sinnúmero de observadores
pregunta:73
Identifique la idea principal del texto.
Para un hispanohablante, las lenguas del mismo origen, es decir las romances o neolatinas (portugués, francés,
italiano, catalán, gallego, rumano, entre otras.), al compartir muchas estructuras y léxico, resultarían más fáciles
para el aprendizaje. Entonces, si la enseñanza se basa en la comparación o contraste gramatical, puede ser
sencillo para una persona adulta que tenga un conocimiento básico de su propia gramática,
aprender una lengua romance. A la hora de estudiar, a un hispanohablante puede resultarle sencilla la
comprensión de otra lengua de la misma familia, como el portugués, por ejemplo, pero, según los especialistas, lo
que parece fácil no suele ser siempre así. La facilidad, sobre todo para la comprensión que se presenta en un
primer momento, hace que no se tengan en cuenta las dificultades porque pasan desapercibidas.
Modificado con fines pedagógicos. Recuperado el 10 de febrero de 2017 en http://bbc.in/2k9ooku
Opciones:
1
Aprender otra lengua de la misma familia puede resultar sencillo, pero según algunos especialistas, lo que parece
fácil no suele ser siempre así
2
La facilidad para la comprensión que se presenta en un primer momento hace que las dificultades pasen
desapercibidas
3
El aprendizaje de lenguas romances o neolatinas resultaría de mayor facilidad para los hablantes nativos de español
4
Si la enseñanza se basa en el contraste gramatical, puede ser sencillo para un adulto con conocimiento básico de
su gramática aprender una lengua romance
Correcta la opción:3
El aprendizaje de lenguas romances o neolatinas resultaría de mayor facilidad para los hablantes
nativos de español
pregunta:74
Identifique la idea principal del texto.
Tal como no saber leer y escribir determinaba el futuro profesional y social de una persona a principios del siglo
XX, hoy no manejar más que el idioma materno es una barrera insoslayable de entrada al mercado laboral para
cualquier joven. Por razones económicas, un segundo idioma se ha vuelto mucho más necesario, como una
herramienta de trabajo. De esta manera comenzó un desafío influenciado por la comunidad hispana de EE.UU.:
integrar el español a las aulas en los distritos de Davis y Granite. No como un segundo idioma, sino en un plan que
contemplara 50 % del tiempo aprendiendo las materias curriculares en inglés y 50
% en español. El programa resultó un éxito y pronto se extendió. Se lanzó a nivel estatal en 2009, cuyo año
académico contó con 1 400 estudiantes. En cinco años, el programa se había ampliado a 25 000 solo en Utah, y
otros estados como Delaware e Indiana comenzarán a implementarlo.
Modificado con fines pedagógicos. Hola, C. (2016) Recuperado el 10 de febrero de 2017 en http://bbc.in/1mIGdEC
Opciones:
1
El desconocimiento de una segunda lengua representa una barrera para los jóvenes que buscan entrar al mundo
laboral
2
A inicios del siglo pasado, el dominio de la lectura y la escritura determinaba el futuro profesional y social de una
persona
3
El programa bilingüe resultó un éxito, de manera que otros estados como Delaware e Indiana comenzarán a
implementarlo
4
Se integró el español a las aulas, no como un segundo idioma, sino en un plan que contemplara 50 % de las
materias curriculares
Correctalaopción:1
El desconocimiento de una segunda lengua representa una barrera para los jóvenes que buscan entrar
al mundo laboral
pregunta:75
Identifique la idea principal del texto.
Puede que las personas que abandonaron su país de origen cuando eran muy pequeñas ya no recuerden su
lengua natal. Sin embargo, los patrones neuronales creados por el idioma que escucharon en sus primeros años
de vida permanecen intactos en su cerebro. Estos patrones se mantienen en el tiempo, incluso si la persona no ha
vuelto a estar en contacto con su primera lengua. Esta huella, dejada por la lengua olvidada, podría facilitarle a
quienes vivieron esta situación, el aprendizaje de su idioma natal en el futuro. En las primeras etapas del desarrollo
de la lengua, los niños aprenden a distinguir -independientemente de qué lengua se trate- qué sonidos son
importantes y significativos. Esta experiencia deja una suerte de representación en el cerebro, que los niños utilizan
para construir su lengua nativa.
Modificado con fines pedagógicos. Recuperado el 10 de febrero de 2017 en http://bbc.in/2l1b5Db
Opciones:
1
La huella dejada por los patrones de la lengua materna facilita el aprendizaje futuro de esta para quienes la han
olvidado
2
Los patrones neuronales creados por la lengua materna jamás se borran del cerebro humano 3
En las primeras etapas del desarrollo de la lengua, los niños aprenden a distinguir los sonidos importantes y
significativos
4
Puede que los niños pequeños que abandonan su país natal no recuerden su lengua materna
Correcta la opción:2
Los patrones neuronales creados por la lengua materna jamás se borran del cerebro humano
pregunta:76
Con base en el texto, identifique la inferencia correcta.
Silencio atronador, muerto viviente, dulce amargura, noche blanca o monstruo hermoso, son ejemplos de oxímoron, una
combinación de dos palabras de significado opuesto que al unirse originan un nuevo sentido. Un estudio español publicado en
la revista NeuroImage, revela que estas figuras literarias generan una intensa actividad en el área frontal izquierda del cerebro.
Según los autores del estudio, "se activa la parte frontal del cerebro y se emplean más recursos de lo habitual en procesar a
nivel cerebral estas expresiones". Para los experimentos, Molinaro y sus colegas crearon varias listas de frases incorrectas,
neutras, oxímoron y pleonasmos. Concretamente, los investigadores han utilizado
"monstruo geográfico" como expresión incorrecta, "monstruo solitario" como expresión neutra, "monstruo hermoso" como
oxímoron, y "monstruo horrible" como pleonasmo. Los resultados muestran que cuanto menos natural es la expresión, más
recursos requiere para ser procesada en la parte frontal izquierda del cerebro.
Modificado con fines pedagógicos. Sanz, E. (s.f.). Recuperado el 17 de junio de 2017 en http://bit.ly/1phnMkO
Opciones:
1
La actividad cerebral se incrementa cuanto menos natural es la expresión, porque requiere más recursos para ser
procesada
2
El oxímoron es la única figura que genera una intensa actividad en el área frontal izquierda del cerebro 3
La parte frontal izquierda del cerebro se activa en procesos de abstracción porque debe construir un sentido
inexistente
4
El oxímoron es una combinación de dos palabras de significado opuesto, que al unirse originan un nuevo sentido
Correcta la opción:3
La parte frontal izquierda del cerebro se activa en procesos de abstracción porque debe construir un
sentido inexistente
pregunta:77
Con base en el texto, identifique la inferencia adecuada.
Hace miles de años, los habitantes de los Andes vienen "domesticando" naturalmente una papa amarga que de
otra manera no sería comestible y que puede conservarse durante décadas. El chuño se prepara en junio y julio, en
pleno invierno del hemisferio sur, cuando llega "el friaje" al altiplano de los Andes y las temperaturas bajan en la
noche hasta los -5 grados. Las comunidades andinas aprovechan el contraste de las temperaturas durante el día y
la noche para "momificar" las papas. En un proceso de liofilización natural, se congelan las papas por las noches y
se deshidratan al sol durante el día, a temperaturas que alcanzan los 18 grados. Una vez cosechadas las papas,
normalmente en el mes de mayo, se llevan hasta las partes planas de la cordillera, llamadas chuñochinapampa.
Esta elaboración artesanal, de unos 20 días de duración (incluyendo el secado), sirve para eliminar todos los
alcaloides que tiene la papa y que le dan el sabor amargo original.
Modificado con fines pedagógicos. Recuperado el 22 de junio de 2017 en http://bbc.in/2slifG5
Opciones:
1
Habitantes de los Andes han domesticado naturalmente una papa amarga que puede conservarse durante décadas
2
Ciertas técnicas ancestrales pueden procesar y conservar alimentos sin usar sustancias químicas o procesos
industriales
3
El proceso artesanal para preparar el chuño (congelación y secado) elimina los alcaloides que dan sabor amargo a
la papa
4
La única propiedad que aporta el proceso ancestral de liofilización de papas amargas es hacerlas comestibles
Correcta la opción:2
Ciertas técnicas ancestrales pueden procesar y conservar alimentos sin usar sustancias químicas o
procesos industriales
pregunta:78
Con base en el texto, identifique la inferencia adecuada.
Los modistos, costureras y sastres que cosen a mano tienen una agudeza visual superior a la de otras personas.
La visión en 3D o estereoscópica de quienes se dedican a la costura es tan afilada como sus propias agujas. Es la
capacidad que tiene nuestro cerebro para recibir la información en 2D que perciben nuestros ojos y transformarla
en una imagen con profundidad o en tres dimensiones. Los profesionales de la costura tienen una precisión 80 %
mayor que las personas de otras profesiones cuando calculan la distancia entre ellos mismos y los objetos que
observan. No está claro si esta actividad afina la visión estereoscópica de las personas o si los sastres se ven
atraídos hacia esta profesión debido a su extraordinaria agudeza visual. Ambas posibilidades, la autoselección y la
experiencia, no son excluyentes, una buena estereovisión es una funcionalidad importante para ser un buen
modisto, a la vez que el entrenamiento regular en tareas que requieren de una buena estereovisión mejora la
agudeza visual.
Modificado con fines pedagógicos. Recuperado el 22 de junio de 2017 en http://bbc.in/2tdmvHZ
Opciones:
1
No está claro si la costura afina la visión estereoscópica o si los sastres eligen su profesión por su eficaz visión 2
Los modistos, las costureras y sastres poseen una visión estereoscópica tan afilada como sus propias agujas 3
El uso de máquinas de coser impide que quienes se dedican a la costura desarrollen la visión estereoscópica 4
Coser a mano podría ser una terapia eficaz para mejorar la agudeza visual de personas con problemas de
estereovisión
Correcta la opción:4
Coser a mano podría ser una terapia eficaz para mejorar la agudeza visual de personas con problemas
de estereovisión
pregunta:79
Identifique el género al que pertenece el fragmento.
¡Pues bien!, yo necesito decirte que te adoro,
decirte que te quiero con todo el corazón;
que es mucho lo que sufro, que es mucho lo que lloro,
que ya no puedo tanto, y al grito en que te imploro,
te imploro y te hablo en nombre de mi última ilusión.
Acuña, M.
Opciones:
1
Épico
2
Narrativo
3
Lírico
4
Dramático
Correcta la opción:3
Lírico
pregunta:80
Identifique el género al que pertenece el fragmento.
En 1823, antes de la partida de las jornadas de Junín y Ayacucho, inconclusa todavía la independencia política,
Andrés Bello proclamaba la independencia espiritual: la primera de sus Silvas americanas es una alocución a la
poesía, “maestra de los pueblos y los reyes”, para que abandone a Europa -luz y miseria- y busque en esta orilla
del Atlántico el aire salubre de que gusta su nativa rustiquez. La forma es clásica; la intención es revolucionaria.
Con la alocución, simbólicamente, iba a encabezar Juan María Gutiérrez nuestra primera grande antología, la
América poética de 1846.
Opciones:
1
Narrativo
2
Ensayístico
3
Dramático
4
Lírico
Correcta la opción:2
Ensayístico pregunta:81
Identifique el género literario al que corresponde el texto.
Volverán las oscuras golondrinas
en tu balcón sus nidos a colgar,
y, otra vez, con el ala a sus cristales jugando llamarán;
pero aquellas que el vuelo refrenaban tu hermosura y mi dicha al contemplar,
aquellas que aprendieron nuestros nombres... ésas... ¡novolverán!
Opciones:
1
Lírico
2
Narrativo
3
Dramático
4
Épico
Correcta la opción:1
Lírico
pregunta:82
¿A qué género literario pertenece el texto?
Entretanto Neptuno advirtió por el ruido tan grande que el mar se agitaba, se
desataba la tormenta y el agua volvía de los profundos abismos
y, gravemente afectado, miró desde lo alto
sacando su plácida cabeza por encima del agua.
Ve por todo el mar la flota deshecha de Eneas,
y a los troyanos atrapados por las olas y la ruina del cielo;
y no se le escaparon al hermano las trampas y la ira de Juno.
Así que llama ante él al Céfiro y al Euro, y así les dice:
«¿A tanto ha llegado el orgullo de la raza vuestra?
¿Ya revolvéis el cielo y la tierra sin mi numen, vientos, y
os atrevéis a levantar moles tangrandes?
Opciones:
1
Épico
2
Lírico
3
Cómico
4
Cuento
Correcta la opción:1
Épico
pregunta:83
Identifique el subgénero al que pertenece el fragmento.
HAMLET:
¡Ah, ya muero, Horacio!
El fuerte veneno señorea mi ánimo.
No viviré para oír las nuevas de Inglaterra,
pero adivino que será elegido rey
Fortinbrás. Le doy mi votoagonizante.
Díselo, junto con todos los sucesos
que me han llevado... El resto es silencio.
[Lanza un hondo suspiro y ] muere.
HORACIO:
Ha estallado un noble pecho. Buenas noches,
buen príncipe; que cánticos de ángeles
te lleven al reposo. ¿Por qué vienen los tambores?
Opciones:
1
Comedia
2
Drama
3
Tragicomedia
4
Tragedia
Correcta la opción:4
Tragedia
pregunta:84
Identifique el subgénero literario al que pertenece el fragmento
No hay extensión más grande que mi herida,
lloro mi desventura y sus conjuntos
y siento más tu muerte que mi vida.
Ando sobre rastrojos de difuntos, y
sin calor de nadie y sin consuelo
voy de mi corazón a mis asuntos.
Temprano levantó la muerte el vuelo,
temprano madrugó la madrugada,
temprano estás rodando por el suelo.
Hernández,M.(s.f.)
Opciones:
1
Oda
2
Égloga
3
Elegía
4
Romance
Correcta la opción:3
Elegía
pregunta:85
Identifique el subgénero al que pertenece el fragmento.
Un pequeño grupo, entre el que está don Américo, acompaña paso a paso el ataúd donde están los restos de
quien en vida fue don Calvo Figueroa, carpintero de profesión, guitarrista por vocación, viudo de la Mercedes, padre
de cuatro hijos varones y seis mujeres, abuelo de nadie sabe cuántos nietos, persona excelente y uno de los
grandes del pueblo, pues medía un metro con 89 centímetros.
Don Américo, en un momento de silencio total en el que hasta los pajaritos dejaron de cantar, dice con voz pausada,
contundente y sonora:
- Pensar que ayer mi compadre estaba lleno de vida… y ahora está perfectamente muerto… pensar que lo único
perfecto que hacemos en la vida es morirnos; eso sí, los mejores, como mi compadre, mueren después de haber
sido buenos, amables y generosos.
Andrade Heymann, J. (2010). Cabizmundo y meditabajo. Pág. 270.
Opciones:
1
Fábula
2
Cuento
3
Epopeya
4
Épica
Correcta la opción:2
Cuento
pregunta:86
Seleccione las características del realismo literario.
1. El hombre como centro de cultura y saber
2. Descripción objetiva de la vida ordinaria
3. Búsqueda de la belleza de las cosas
4. Enjuiciamiento crítico de la sociedad
5. Presentación de personajes de distintas clases sociales
6. Conocimiento de autores griegos y latinos
Opciones:
1
1, 2, 4
2
1, 3, 6
3
2, 4, 5
4
3, 5, 6
Correctalaopción:3
2, 4, 5
pregunta:87
Seleccione las características determinantes de una obra simbolista.
1. Emplea de forma recurrente la figura de la imagen
2. Se describe de forma minuciosa la realidad
3. Se elude nombrar directamente los objetos
4. Emplea de forma recurrente la figura del hipérbaton
Opciones:
1
1, 3
2
1, 4
3
2, 3
4
2, 4
Correctalaopción:1
1, 3
pregunta:88
Relacione la escuela literaria con su característica.
Escuela Característica
1. Romanticismo a) La libertad de pensamiento y la idealización de la naturaleza fueron las fuentes de
inspiración
2. Clasicismo b) Exalta el individualismo. Hay cambios en los temas y sus formas. Se caracterizó por
retomar los valores de la cultura grecorromana
3. Renacimiento c) Las composiciones se caracterizaron por ser equilibradas, sobrias y armónicas
Opciones:
1
1a, 2c, 3b
2
1b, 2a, 3c
3
1b, 2c, 3a
4
1c, 2a, 3b
Correcta la opción:1
1a, 2c, 3b
pregunta:89
Identifique la escuela literaria que se destaca por su intención renovadora, de avance y exploración.
Opciones:
1
Clasicismo
2
Romanticismo
3
Vanguardismo
4
Realismo
Correcta la opción:3
Vanguardismo
pregunta:90
La escuela literaria del modernismo se caracteriza por:
Opciones:
1
exceder los límites aceptados como norma cultural europea de inicios del siglo XX 2
priorizar los sentimientos del individuo como respuesta al racionalismo y clasicismo europeo 3
surgir en América Latina a finales del siglo XIX bajo influencia del parnasianismo y simbolismo 4
reproducir la realidad de forma casi documental en todos los aspectos, tanto sublimes como vulgares
Correcta la opción:3
surgir en América Latina a finales del siglo XIX bajo influencia del parnasianismo y simbolismo
pregunta:91
Identifique al autor del fragmento.
Revestir sabe el vino los más sórdidos antros
De un milagroso lujo,
Y hace surgir más de un pórtico fabuloso
Entre el oro de su rojo vapor,
Como el sol que se pone en un cielo nublado
Agranda el opio aquello que no tolera límites, Lo
ilimitado alarga,
El tiempo profundiza, los deleites ahonda, Y
de placer triste y oscuro,
Anega y colma al alma rebasada.
Mas todo eso no vale el prodigio terrible
De tus ojos, de tus verdes ojos,
Lagos donde mi alma tiembla y se ve invertida…
Llegan mis sueños en tropel
Para abrevar en esos dos abismos amargos
Mas todo esto no vale prodigio terrible
De tu mordiente saliva,
Que sume en el olvido a mi alma impenitente Y,
el vértigo arrastrando,
La trae desfallecida a orillas de la muerte.
Opciones:
1
Rubén Darío
2
Paul Verlaine
3
Fernando Pessoa
4
Charles Baudelaire
Correcta la opción:4
Charles Baudelaire
pregunta:92
Seleccione las obras adscritas al clasicismo literario.
1. Soledades
2. LasGeórgicas
3. Las Bucólicas
4. Los viajes de Gulliver
Opciones:
1
1, 3
2
1, 4
3
2, 3
4
2, 4
Correctalaopción:3
2, 3
pregunta:93
Elija las obras adscritas al barroco literario.
1. La vida del Buscón
2. Robinson Crusoe
3. Fábula de Polifemo y Galatea
4. Aventuras del Capitán Singleton
Opciones:
1
1, 3
2
1, 4
3
2, 3
4
2, 4
Correctalaopción:1
1, 3
pregunta:94
Relacione la escuela literaria con su obra representativa.
Escuela Obra
Opciones:
1
1a, 2b, 3c, 4d
2
1b, 2a, 3d, 4c
3
1c, 2d, 3a, 4b
4
1d, 2c, 3b, 4a
1. Culteranismo a) Volverán las oscuras golondrinas
2. Romanticismo b) Polifemo y Galatea
3. Realismo c) La tía Tula
4. Modernismo d) Pepita Jiménez
Correcta la opción:2
1b, 2a, 3d, 4c
pregunta:95
Relacione el autor con la corriente literaria a la que pertenece.
Autor Corriente
Opciones:
1
1a, 2d, 3b, 4c
2
1b, 2a, 3c, 4d
3
1c, 2b, 3d, 4a
4
1d, 2c, 3a, 4b
Correcta la opción:4
1d, 2c, 3a, 4b
pregunta:96
¿A qué corriente literaria pertenece el fragmento?
En lo físico, Grandet era hombre de cinco pies, rechoncho, cuadrado, con unas pantorrillas de doce pulgadas de
circunferencia, grandes rótulas y anchas espaldas; su cara era redonda, curtida y marcada por la viruela; su barba
era recta, sus labios no ofrecían ninguna sinuosidad y sus dientes eran blancos; sus ojos tenían la expresión
tranquila y devoradora que el pueblo atribuye al basilisco; su frente, llena de arrugas transversales, no carecía de
significativas protuberancias; y sus cabellos, rubios y blancos, eran de color plata y oro, al decir de algunas gentes
que no conocían la gravedad que podía tener el hecho de gastar una broma al señor
1. Federico García Lorca
2. Tristan Tzara
3. Vicente HuidobroFernández
4. Guillermo de Torre
a) Creacionismo
b) Ultraísmo
c) Dadaísmo
d) Surrealismo
Grandet. Su nariz, gorda por la punta, sostenía un lobanillo veteado que, según decía el vulgo, y no sin razón,
estaba lleno de malicia. Esta cara anunciaba esa astucia peligrosa, esa fría probidad y ese egoísmo del hombre
acostumbrado a concentrar sus sentimientos en el único ser que le fue siempre querido, en su hija Eugenia, en su
única heredera. Por otra parte, la actitud, los modales, el paso, todo en él confirmaba esa creencia en sí que da la
costumbre de ver que se sale siempre airoso en sus empresas; así, pues, aunque el señor Grandet era, en
apariencia, hombre de costumbres sencillas y afeminadas, tenía un carácter dehierro.
Balzac. (1833) Eugenia Grandet. Editorial No Books.
Opciones:
1
Simbolismo
2
Naturalismo 3
Realismo
4
Romanticismo
Correcta la opción:3
Realismo
pregunta:97
Identifique la corriente literaria a la que pertenece el fragmento.
El albatros (fragmento)
Este alado viajero, ¡qué inútil y qué débil! Él,
otrora tan bello, ¡qué feo y qué grotesco!
¡Este quema su pico, sádico, con la pipa,
Aquél, mima cojeando al planeador inválido!
El poeta es igual a este señor del nublo, Que
habita la tormenta y ríe del ballestero.
Exiliado en la tierra, sufriendo el griterío, Sus
alas de gigante le impiden caminar.
Baudelaire, C. (1851) El Albatros.
Opciones:
1
Barroco
2
Simbolismo
3
Culteranismo 4
Conceptismo
Correcta la opción:2
Simbolismo
pregunta:98
Relacione el autor con la corriente literaria correspondiente.
Autor Corriente
1.
André
Breton a) Expresionismo
Opciones:
1
1a, 2b, 3c
2
1a, 2c, 3b
3
1b, 2a, 3c
4
1c, 2a, 3b
2.
Franz Kafka b) Dadaísmo
3.
Hugo Ball c) Surrealismo
Correcta la opción:4
1c, 2a, 3b
pregunta:99
Con base en el fragmento, identifique la corriente a la que pertenece el poema.
Vehemente dios de una raza de acero,
automóvil ebrio de espacio,
que piafas y te estremeces de angustia
tascando el freno con estridentes dientes…
Formidable monstruo japonés
de ojos de fragua,
nutrido de llama
y de aceites minerales,
ávido de horizontes y presas siderales…
¡yo desencadeno tu corazón que golpea diabólicamente,
desencadeno tus gigantescos neumáticos,
para la danza que sabes danzar
en los blancos caminos de todo el mundo!
Marinetti, F. T. El automóvil de carrera. Recuperado el 28 de agosto de 2014 en http://bit.ly/YpoIxj
Opciones:
1
Surrealismo
2
Futurismo
3
Expresionismo
4
Dadaísmo
Correcta la opción:2
Futurismo
pregunta:100
Elija los poemas producidos durante el periodo modernista ecuatoriano.
1. El alma en los labios
2. A unos ojos hermosos
3. Primavera mística y lunar
4. A Carmen
5. Canto a Bolívar
6. Romanza de las horas
Opciones:
1
1, 2, 5
2
1, 3, 6
3
2, 4, 5
4
3, 4, 6
Correctalaopción:2
1, 3, 6
pregunta:101
Relacione la autora con su obra.
Opciones:
1
1a, 2c, 3b
2
1b, 2c, 3a
3
1c, 2a, 3b
4
1c, 2b, 3a
Autora Obra
1. Alicia Yánez Cossío a) Así se hace una mamá
2. Teresa León de Noboa b) Los gatos literatos
3. Catalina Sojos c) Bruna, soroche y los tíos
Correcta la opción:4
1c, 2b, 3a
pregunta:102
Relacione el autor con su obra.
Opciones:
1
1a, 2b, 3c
2
1a, 2c, 3b
3
1b, 2c, 3a
4
1c, 2b, 3a
Autor Obra
1. Humberto Salvador a) Entre Marx y una mujer desnuda
2. Jorge Velasco Mackenzie b) El rincón de los justos
3. Jorge Enrique Adoum c) En la ciudad he perdido una novela
Correcta la opción:4
1c, 2b, 3a
pregunta:103
Relacione la obra literaria ecuatoriana con su autor.
Opciones:
1
1a, 2c, 3b, 4d
2
1b, 2a, 3d, 4c
3
1d, 2a, 3b, 4c
4
1d, 2c, 3b, 4a
Correcta la opción:4
1d, 2c, 3b, 4a
pregunta:104
Elija las obras del escritor Albert Camus.
1. El proceso
2. La náusea
3. La peste
4. El extranjero
Opciones:
Obra Autor
1. Los Sangurimas a) EliécerCárdenas
2. La emancipada b) Luis A. Martínez
3. A la Costa c) Miguel Riofrío
4. Polvo y ceniza d) José de la Cuadra
1
1, 2
2
1, 3
3
2, 4
4
3, 4
Correctalaopción:4
3, 4
pregunta:105
Elija las obras del autor John Ronald Reuel Tolkien.
1. Harry Potter
2. El Señor de los Anillos
3. Las Crónicas de Narnia
4. El Hobbit
Opciones:
1
1, 3
2
1, 4
3
2, 3
4
2, 4
Correctalaopción:4
2, 4
pregunta:106
Identifique el autor al que hace referencia el texto.
La mayoría de sus narraciones se desarrollan en San Martín, una ciudad ficticia al borde de un río. Entre sus obras
emblemáticas se encuentran: La vida breve (1950), donde Brausen, el protagonista, enclaustrado en su
apartamento, construye una ficción que le permitirá ser otro; Juntacadáveres (1964), que relata un fragmento de la
vida de Larsen cuando abre un burdel en San Martín; El astillero (1961) que se considera su obra maestra, donde
Larsen se convierte en un ejecutivo muy bien remunerado; mientras que Dejemos hablar al viento (1979) representa
el fracaso del hombre.
Modificado con fines pedagógicos. Becerra, E. (s/f). Historia del otro lado. Recuperado el 2 de abril de 2017 en https://goo.gl/agxd3a
Opciones:
1
José Donoso
2
Octavio Paz
3
Juan Onetti
4
Carlos Fuentes
Correcta la opción:3
Juan Onetti
pregunta:107
Identifique la clase de rima que hay en la estrofa.
Bajo el dosel de gigantesca roca
yace el titán, cual Cristo en el Calvario,
marmóreo, indiferente y solitario,
sin que brote el gemido de su boca.
Del Casal, J. Prometeo.
Opciones:
1
Continua
2
Cruzada
3
Abrazada
4
Pareada
Correcta la opción:3
Abrazada
pregunta:108
Identifique la clase de rima que se aplica en la estrofa.
Dolores, costurera de mi casa
añosa de mi casa, vieja amiga;
era tu corazón crujiente miga
de pan; eran tus ojos lenta brasa.
Panero, L. (s.f.). Recuperado el 13 de agosto de 2013 en http://roble.pntic.mec.es/msanto1/lengua/estrofas.htm
Opciones:
1
Asonante
2
Consonante
3
Ritmo
4
Libre
Correcta la opción:2
Consonante
pregunta:109
Identifique la clase de rima.
Fue en un poniente mágico de púrpura y oros:
con música de brisas en los pinos sonoros,
rítmicas desfilaban las horas, al ocaso,
tal una ronda griega cincelada en un vaso
Silva, M. (s.f.). La investidura. Recuperado el 29 de marzo de 2017 en http://bit.ly/2nfxoT5
Opciones:
1
Pareada
2
Continua
3
Abrazada
4
Encadenada
Correcta la opción:1
Pareada
pregunta:110
Identifique la clase de rima.
Largo espectro de plata conmovida
el viento de la noche suspirando,
abrió con mano gris mi vieja herida y
se alejó: yo estaba deseando.
Llaga de amor que me dará la vida
perpetua sangre y pura luz brotando.
García, F. (s.f.). Largo espectro de plata conmovida. Recuperado el 29 de marzo de 2017 en http://bit.ly/2nfAvu9
Opciones:
1
Encadenada
2
Pareada
3
Continua
4
Abrazada
Correcta la opción:1
Encadenada
pregunta:111
Identifique la clase de rima.
Hombres necios que acusáis a
la mujer sin razón,
sin ver que sois la ocasión
de lo mismo que culpáis
De la Cruz, I. (s.f.). Redondillas. Recuperado el 29 de marzo de 2017 en http://bit.ly/2mOva1I
Opciones:
1
Pareada
2
Continua
3
Encadenada
4
Abrazada
Correcta la opción:4
Abrazada
pregunta:112
Identifique la métrica del verso en negrita.
Un mozo trae por un sendero
sus herramientas y su morral:
otro con caites y sin sombrero
busca una vaca con su ternero
para ordeñarla junto al corral.
Rubén Darío. Del trópico. Recuperado el 10 de febrero de 2016 en http://bit.ly/1O3YKRL
Opciones:
1
Decasílabo
2
Octosílabo
3
Endecasílabo
4
Dodecasílabo
Correcta la opción:1
Decasílabo
pregunta:113
¿Qué tipo de versos tiene esta estrofa?
Poesía XLIV
Tiene el leopardo un abrigo
en su monte seco y pardo
yo tengo más que el leopardo
porque tengo un buen amigo
Martí, J. (1891). Versos sencillos. Recuperado el 13 de agosto de 2013 en http://www.damisela.com/literatura/pais/cuba/autores/marti/sencillo/xliv.htm
Opciones:
1
Eneasílabos
2
Octosílabos
3
Endecasílabos
4
Hexasílabos
Correcta la opción:2
Octosílabos
pregunta:114
Identifique el verso octosílabo de la autora Concha Benito.
Opciones:
1
Dime tú, ahora
2
De tu cabeza de águila afiebrada
3
Del fauno ingrato que un día 4
De tu seso enjambrado
Correcta la opción:3
Del fauno ingrato que un día
pregunta:115
Con base en el fragmento, identifique la métrica de la estrofa.
¡Oh dulces prendas, por mi mal halladas,
dulces y alegres cuando Dios quería!
Juntas estáis en la memoria mía,
y con ella en mi muerte conjuradas.
De la Vega, G. Soneto X. Recuperado el 28 de agosto de 2014 en http://bit.ly/1C7zdVo
Opciones:
1
Endecasílabo
2
Decasílabo
3
Eneasílabo
4
Dodecasílabo
Correcta la opción:1
Endecasílabo
pregunta:116
Identifique el tipo de métrica presente en la estrofa.
Era una tarde de enero; el
sol casi se ocultaba,
y las brisas dulcemente
gemían entre las ramas.
Darío, R. Romance. Recuperado el 15 de agosto de 2014 en http://bit.ly/1oYxIom
Opciones:
1
Heptasílabo
2
Decasílabo
3
Octosílabo
4
Eneasílabo
Correcta la opción:3
Octosílabo
pregunta:117
Relacione el periodo de la literatura ecuatoriana con su obra representativa.
Opciones:
1
1a, 2d, 3c, 4b
2
1b, 2a, 3c, 4d
3
1d, 2a, 3b, 4c
4
1d, 2b, 3a, 4c
Periodo Obra
1. Precolonial a) La conquista de Menorca
2. Colonial b) Oda a Miñarica
3. Independentista c) A mis enemigos
4. Republicano d) Atahualpa Huañui
Correcta la opción:3
1d, 2a, 3b, 4c
pregunta:118
¿En qué verso se usa el polisíndeton?
Opciones:
1
Y el odio y el furor de los tiranos 2
En tierra, en humo, en polvo, en sombra, en nada
3
El huracán aullaba como mastín de caza
4
El Chimborazo alzaba su cabeza de abuelo
Correcta la opción:1
Y el odio y el furor de los tiranos
pregunta:119
Elija los versos que contienen retruécano.
1. Cortante como una daga
2. Voy a prenderme fuego sin llamas, ahora que no me llamas ni para pedirme fuego
3. Ni dice lo que siente ni siente lo que dice
4. Millones de ideas lo atacaban
5. No rezo para vivir ni vivo para rezar
6. Es tan corto el amor y tan largo el olvido
Opciones:
1
1, 3, 4
2
1, 4, 6
3
2, 3, 5
4
2, 5, 6
Correctalaopción:3
2, 3, 5
pregunta:120
Con base en el fragmento, identifique la figura literaria presente en la estrofa.
Tum tum, ¿quién es?
una rosa y un clavel
abre la muralla.
Tum tum, ¿quién es?
el sable del coronel
cierra la muralla.
Guillén, N. La muralla. Recuperado el 28 de agosto de 2014 en http://bit.ly/VOzuME
Opciones:
1
Asindetón
2
Onomatopeya
3
Elipsis
4
Síncopa
Correcta la opción:2
Onomatopeya

Mais conteúdo relacionado

Mais procurados

Crónica literaria y perspectivismo en Crónica de una muerte anunciada
Crónica literaria y perspectivismo en Crónica de una muerte anunciadaCrónica literaria y perspectivismo en Crónica de una muerte anunciada
Crónica literaria y perspectivismo en Crónica de una muerte anunciadaJuan Carlos Reinaldos
 
El gato negro de Edgar Allan Poe
 El gato negro de Edgar Allan Poe  El gato negro de Edgar Allan Poe
El gato negro de Edgar Allan Poe Aurelio Mizvic
 
Semana 11 Dif. estructurales mito y leyenda
Semana 11 Dif. estructurales mito y leyenda Semana 11 Dif. estructurales mito y leyenda
Semana 11 Dif. estructurales mito y leyenda SW México Preparatoria
 
OSVALDO HURTADO LARREA
OSVALDO HURTADO LARREAOSVALDO HURTADO LARREA
OSVALDO HURTADO LARREAwagner rosero
 
Carta de la esclavitud y jose flores
Carta de la esclavitud y jose floresCarta de la esclavitud y jose flores
Carta de la esclavitud y jose floresSebastian Barbosa
 
Análisis de ALGO MUY GRAVE VA A SUCEDER EN ESTE PUEBLO
Análisis de ALGO MUY GRAVE VA A SUCEDER EN ESTE PUEBLOAnálisis de ALGO MUY GRAVE VA A SUCEDER EN ESTE PUEBLO
Análisis de ALGO MUY GRAVE VA A SUCEDER EN ESTE PUEBLOMaria Dum
 
democracia política y democracia social
democracia política y democracia socialdemocracia política y democracia social
democracia política y democracia socialGhala Ramírez
 
Actividad gato negro cun 1 machado
Actividad gato negro cun 1 machadoActividad gato negro cun 1 machado
Actividad gato negro cun 1 machadoluceroazul
 
Analisis heraldos negros original
Analisis heraldos negros originalAnalisis heraldos negros original
Analisis heraldos negros originalNombre Apellidos
 
Génesis y Popol Vuh diferencias y semejanzas.
Génesis y Popol Vuh diferencias y semejanzas.Génesis y Popol Vuh diferencias y semejanzas.
Génesis y Popol Vuh diferencias y semejanzas.willmarojeda
 
Productos con tarifa 0% y12%
Productos con tarifa 0% y12%Productos con tarifa 0% y12%
Productos con tarifa 0% y12%NathalySerrano1
 
El tiempo y los calendarios
El tiempo y los calendariosEl tiempo y los calendarios
El tiempo y los calendariosdarwinprm
 
Lectura expresión oral y escrita
Lectura expresión oral y escritaLectura expresión oral y escrita
Lectura expresión oral y escritaapoyateenmi12
 
Democracia, Participación ,Dialéctica ,Edad: Antigua, medieval, Moderna, Con...
Democracia, Participación ,Dialéctica ,Edad: Antigua,  medieval, Moderna, Con...Democracia, Participación ,Dialéctica ,Edad: Antigua,  medieval, Moderna, Con...
Democracia, Participación ,Dialéctica ,Edad: Antigua, medieval, Moderna, Con...Doris Molero
 
Analisis Literario de Los Cachorros
Analisis Literario de Los CachorrosAnalisis Literario de Los Cachorros
Analisis Literario de Los CachorrosLuis Morales
 
Historia 1-bgu
Historia 1-bguHistoria 1-bgu
Historia 1-bguLulyMaty1
 

Mais procurados (20)

Crónica literaria y perspectivismo en Crónica de una muerte anunciada
Crónica literaria y perspectivismo en Crónica de una muerte anunciadaCrónica literaria y perspectivismo en Crónica de una muerte anunciada
Crónica literaria y perspectivismo en Crónica de una muerte anunciada
 
El gato negro de Edgar Allan Poe
 El gato negro de Edgar Allan Poe  El gato negro de Edgar Allan Poe
El gato negro de Edgar Allan Poe
 
Semana 11 Dif. estructurales mito y leyenda
Semana 11 Dif. estructurales mito y leyenda Semana 11 Dif. estructurales mito y leyenda
Semana 11 Dif. estructurales mito y leyenda
 
OSVALDO HURTADO LARREA
OSVALDO HURTADO LARREAOSVALDO HURTADO LARREA
OSVALDO HURTADO LARREA
 
Carta de la esclavitud y jose flores
Carta de la esclavitud y jose floresCarta de la esclavitud y jose flores
Carta de la esclavitud y jose flores
 
Gobierno de vicente rocafuerte
Gobierno de vicente rocafuerteGobierno de vicente rocafuerte
Gobierno de vicente rocafuerte
 
Plata y bronce
Plata y broncePlata y bronce
Plata y bronce
 
Análisis de ALGO MUY GRAVE VA A SUCEDER EN ESTE PUEBLO
Análisis de ALGO MUY GRAVE VA A SUCEDER EN ESTE PUEBLOAnálisis de ALGO MUY GRAVE VA A SUCEDER EN ESTE PUEBLO
Análisis de ALGO MUY GRAVE VA A SUCEDER EN ESTE PUEBLO
 
democracia política y democracia social
democracia política y democracia socialdemocracia política y democracia social
democracia política y democracia social
 
Actividad gato negro cun 1 machado
Actividad gato negro cun 1 machadoActividad gato negro cun 1 machado
Actividad gato negro cun 1 machado
 
Los Saraguros
Los SaragurosLos Saraguros
Los Saraguros
 
Analisis heraldos negros original
Analisis heraldos negros originalAnalisis heraldos negros original
Analisis heraldos negros original
 
Génesis y Popol Vuh diferencias y semejanzas.
Génesis y Popol Vuh diferencias y semejanzas.Génesis y Popol Vuh diferencias y semejanzas.
Génesis y Popol Vuh diferencias y semejanzas.
 
Todos santos beni
Todos santos beniTodos santos beni
Todos santos beni
 
Productos con tarifa 0% y12%
Productos con tarifa 0% y12%Productos con tarifa 0% y12%
Productos con tarifa 0% y12%
 
El tiempo y los calendarios
El tiempo y los calendariosEl tiempo y los calendarios
El tiempo y los calendarios
 
Lectura expresión oral y escrita
Lectura expresión oral y escritaLectura expresión oral y escrita
Lectura expresión oral y escrita
 
Democracia, Participación ,Dialéctica ,Edad: Antigua, medieval, Moderna, Con...
Democracia, Participación ,Dialéctica ,Edad: Antigua,  medieval, Moderna, Con...Democracia, Participación ,Dialéctica ,Edad: Antigua,  medieval, Moderna, Con...
Democracia, Participación ,Dialéctica ,Edad: Antigua, medieval, Moderna, Con...
 
Analisis Literario de Los Cachorros
Analisis Literario de Los CachorrosAnalisis Literario de Los Cachorros
Analisis Literario de Los Cachorros
 
Historia 1-bgu
Historia 1-bguHistoria 1-bgu
Historia 1-bgu
 

Semelhante a LENGUA Y LITERATURA (1).pdf

2014 queescondemipueblosegundobasico 1
2014 queescondemipueblosegundobasico 12014 queescondemipueblosegundobasico 1
2014 queescondemipueblosegundobasico 1Tia Vib
 
El lagarto proyecto de aprendizaje
El lagarto proyecto de aprendizajeEl lagarto proyecto de aprendizaje
El lagarto proyecto de aprendizajeValentin Flores
 
Lengua y-literatura8-10 egb
Lengua y-literatura8-10 egbLengua y-literatura8-10 egb
Lengua y-literatura8-10 egbchristian1342
 
2°-BÁSICO.GUÍA-EVALUADA.-LENGUAJE.pdf
2°-BÁSICO.GUÍA-EVALUADA.-LENGUAJE.pdf2°-BÁSICO.GUÍA-EVALUADA.-LENGUAJE.pdf
2°-BÁSICO.GUÍA-EVALUADA.-LENGUAJE.pdfMaria Carvajal
 
P L A N D E R E D A C C I O N P A R A J U N I O
P L A N  D E  R E D A C C I O N  P A R A  J U N I OP L A N  D E  R E D A C C I O N  P A R A  J U N I O
P L A N D E R E D A C C I O N P A R A J U N I ORAUL POHL
 
Plan de redaccion para junio
Plan de redaccion para junioPlan de redaccion para junio
Plan de redaccion para junioRaul Otto Pohl
 
Laboratorios y rally académico
Laboratorios y   rally académicoLaboratorios y   rally académico
Laboratorios y rally académicoLissethe Rodriguez
 
4.-CUADERNILLO_cuarto_grado._ok._(1)[1].docx
4.-CUADERNILLO_cuarto_grado._ok._(1)[1].docx4.-CUADERNILLO_cuarto_grado._ok._(1)[1].docx
4.-CUADERNILLO_cuarto_grado._ok._(1)[1].docxYuridiaGmez1
 
4. cuadernillo cuarto grado. ok. (1)
4. cuadernillo cuarto grado. ok. (1)4. cuadernillo cuarto grado. ok. (1)
4. cuadernillo cuarto grado. ok. (1)Jurisdiccion
 
3º SOY LECTOR PART1- MD EDUCATIVO (1).pdf
3º SOY LECTOR PART1- MD  EDUCATIVO (1).pdf3º SOY LECTOR PART1- MD  EDUCATIVO (1).pdf
3º SOY LECTOR PART1- MD EDUCATIVO (1).pdfMartinZorrillatoledo2
 
4to grado bimestre 1(11-12)
4to grado   bimestre 1(11-12)4to grado   bimestre 1(11-12)
4to grado bimestre 1(11-12)bebe1ooo
 
Biología preparatoria tec m
Biología preparatoria tec mBiología preparatoria tec m
Biología preparatoria tec mEducaciontodos
 
Biología preparatoria tec m
Biología preparatoria tec mBiología preparatoria tec m
Biología preparatoria tec mMaestros Online
 
Prueba de entrenamiento 1 indoamerica
Prueba de entrenamiento 1 indoamericaPrueba de entrenamiento 1 indoamerica
Prueba de entrenamiento 1 indoamericaJavier Martinez
 

Semelhante a LENGUA Y LITERATURA (1).pdf (20)

2014 queescondemipueblosegundobasico 1
2014 queescondemipueblosegundobasico 12014 queescondemipueblosegundobasico 1
2014 queescondemipueblosegundobasico 1
 
El lagarto proyecto de aprendizaje
El lagarto proyecto de aprendizajeEl lagarto proyecto de aprendizaje
El lagarto proyecto de aprendizaje
 
Falsos amigos
Falsos amigosFalsos amigos
Falsos amigos
 
Lengua y-literatura8-10 egb
Lengua y-literatura8-10 egbLengua y-literatura8-10 egb
Lengua y-literatura8-10 egb
 
Lengua y-literatura8-10 egb
Lengua y-literatura8-10 egbLengua y-literatura8-10 egb
Lengua y-literatura8-10 egb
 
2° básico.guía-evaluada.-lenguaje
2° básico.guía-evaluada.-lenguaje2° básico.guía-evaluada.-lenguaje
2° básico.guía-evaluada.-lenguaje
 
2°-BÁSICO.GUÍA-EVALUADA.-LENGUAJE.pdf
2°-BÁSICO.GUÍA-EVALUADA.-LENGUAJE.pdf2°-BÁSICO.GUÍA-EVALUADA.-LENGUAJE.pdf
2°-BÁSICO.GUÍA-EVALUADA.-LENGUAJE.pdf
 
P L A N D E R E D A C C I O N P A R A J U N I O
P L A N  D E  R E D A C C I O N  P A R A  J U N I OP L A N  D E  R E D A C C I O N  P A R A  J U N I O
P L A N D E R E D A C C I O N P A R A J U N I O
 
Plan de redaccion para junio
Plan de redaccion para junioPlan de redaccion para junio
Plan de redaccion para junio
 
Laboratorios y rally académico
Laboratorios y   rally académicoLaboratorios y   rally académico
Laboratorios y rally académico
 
4.-CUADERNILLO_cuarto_grado._ok._(1)[1].docx
4.-CUADERNILLO_cuarto_grado._ok._(1)[1].docx4.-CUADERNILLO_cuarto_grado._ok._(1)[1].docx
4.-CUADERNILLO_cuarto_grado._ok._(1)[1].docx
 
4. cuadernillo cuarto grado. ok. (1)
4. cuadernillo cuarto grado. ok. (1)4. cuadernillo cuarto grado. ok. (1)
4. cuadernillo cuarto grado. ok. (1)
 
3º SOY LECTOR PART1- MD EDUCATIVO (1).pdf
3º SOY LECTOR PART1- MD  EDUCATIVO (1).pdf3º SOY LECTOR PART1- MD  EDUCATIVO (1).pdf
3º SOY LECTOR PART1- MD EDUCATIVO (1).pdf
 
Actividades anaya
Actividades anayaActividades anaya
Actividades anaya
 
Actividades anaya
Actividades anayaActividades anaya
Actividades anaya
 
4to grado bimestre 1(11-12)
4to grado   bimestre 1(11-12)4to grado   bimestre 1(11-12)
4to grado bimestre 1(11-12)
 
Preguntas de 2º al 7º de basica.pdf amarillas
Preguntas de 2º al 7º de basica.pdf amarillasPreguntas de 2º al 7º de basica.pdf amarillas
Preguntas de 2º al 7º de basica.pdf amarillas
 
Biología preparatoria tec m
Biología preparatoria tec mBiología preparatoria tec m
Biología preparatoria tec m
 
Biología preparatoria tec m
Biología preparatoria tec mBiología preparatoria tec m
Biología preparatoria tec m
 
Prueba de entrenamiento 1 indoamerica
Prueba de entrenamiento 1 indoamericaPrueba de entrenamiento 1 indoamerica
Prueba de entrenamiento 1 indoamerica
 

Último

AFICHE EL MANIERISMO HISTORIA DE LA ARQUITECTURA II
AFICHE EL MANIERISMO HISTORIA DE LA ARQUITECTURA IIAFICHE EL MANIERISMO HISTORIA DE LA ARQUITECTURA II
AFICHE EL MANIERISMO HISTORIA DE LA ARQUITECTURA IIIsauraImbrondone
 
BIOMETANO SÍ, PERO NO ASÍ. LA NUEVA BURBUJA ENERGÉTICA
BIOMETANO SÍ, PERO NO ASÍ. LA NUEVA BURBUJA ENERGÉTICABIOMETANO SÍ, PERO NO ASÍ. LA NUEVA BURBUJA ENERGÉTICA
BIOMETANO SÍ, PERO NO ASÍ. LA NUEVA BURBUJA ENERGÉTICAÁngel Encinas
 
Registro Auxiliar - Primaria 2024 (1).pptx
Registro Auxiliar - Primaria  2024 (1).pptxRegistro Auxiliar - Primaria  2024 (1).pptx
Registro Auxiliar - Primaria 2024 (1).pptxFelicitasAsuncionDia
 
INSTRUCCION PREPARATORIA DE TIRO .pptx
INSTRUCCION PREPARATORIA DE TIRO   .pptxINSTRUCCION PREPARATORIA DE TIRO   .pptx
INSTRUCCION PREPARATORIA DE TIRO .pptxdeimerhdz21
 
Caja de herramientas de inteligencia artificial para la academia y la investi...
Caja de herramientas de inteligencia artificial para la academia y la investi...Caja de herramientas de inteligencia artificial para la academia y la investi...
Caja de herramientas de inteligencia artificial para la academia y la investi...Lourdes Feria
 
SELECCIÓN DE LA MUESTRA Y MUESTREO EN INVESTIGACIÓN CUALITATIVA.pdf
SELECCIÓN DE LA MUESTRA Y MUESTREO EN INVESTIGACIÓN CUALITATIVA.pdfSELECCIÓN DE LA MUESTRA Y MUESTREO EN INVESTIGACIÓN CUALITATIVA.pdf
SELECCIÓN DE LA MUESTRA Y MUESTREO EN INVESTIGACIÓN CUALITATIVA.pdfAngélica Soledad Vega Ramírez
 
PIAR v 015. 2024 Plan Individual de ajustes razonables
PIAR v 015. 2024 Plan Individual de ajustes razonablesPIAR v 015. 2024 Plan Individual de ajustes razonables
PIAR v 015. 2024 Plan Individual de ajustes razonablesYanirisBarcelDelaHoz
 
Valoración Crítica de EEEM Feco2023 FFUCV
Valoración Crítica de EEEM Feco2023 FFUCVValoración Crítica de EEEM Feco2023 FFUCV
Valoración Crítica de EEEM Feco2023 FFUCVGiustinoAdesso1
 
ACERTIJO DE POSICIÓN DE CORREDORES EN LA OLIMPIADA. Por JAVIER SOLIS NOYOLA
ACERTIJO DE POSICIÓN DE CORREDORES EN LA OLIMPIADA. Por JAVIER SOLIS NOYOLAACERTIJO DE POSICIÓN DE CORREDORES EN LA OLIMPIADA. Por JAVIER SOLIS NOYOLA
ACERTIJO DE POSICIÓN DE CORREDORES EN LA OLIMPIADA. Por JAVIER SOLIS NOYOLAJAVIER SOLIS NOYOLA
 
Plan Refuerzo Escolar 2024 para estudiantes con necesidades de Aprendizaje en...
Plan Refuerzo Escolar 2024 para estudiantes con necesidades de Aprendizaje en...Plan Refuerzo Escolar 2024 para estudiantes con necesidades de Aprendizaje en...
Plan Refuerzo Escolar 2024 para estudiantes con necesidades de Aprendizaje en...Carlos Muñoz
 
MAYO 1 PROYECTO día de la madre el amor más grande
MAYO 1 PROYECTO día de la madre el amor más grandeMAYO 1 PROYECTO día de la madre el amor más grande
MAYO 1 PROYECTO día de la madre el amor más grandeMarjorie Burga
 
Qué es la Inteligencia artificial generativa
Qué es la Inteligencia artificial generativaQué es la Inteligencia artificial generativa
Qué es la Inteligencia artificial generativaDecaunlz
 
Programacion Anual Matemática4 MPG 2024 Ccesa007.pdf
Programacion Anual Matemática4    MPG 2024  Ccesa007.pdfProgramacion Anual Matemática4    MPG 2024  Ccesa007.pdf
Programacion Anual Matemática4 MPG 2024 Ccesa007.pdfDemetrio Ccesa Rayme
 
ACUERDO MINISTERIAL 078-ORGANISMOS ESCOLARES..pptx
ACUERDO MINISTERIAL 078-ORGANISMOS ESCOLARES..pptxACUERDO MINISTERIAL 078-ORGANISMOS ESCOLARES..pptx
ACUERDO MINISTERIAL 078-ORGANISMOS ESCOLARES..pptxzulyvero07
 

Último (20)

Medición del Movimiento Online 2024.pptx
Medición del Movimiento Online 2024.pptxMedición del Movimiento Online 2024.pptx
Medición del Movimiento Online 2024.pptx
 
AFICHE EL MANIERISMO HISTORIA DE LA ARQUITECTURA II
AFICHE EL MANIERISMO HISTORIA DE LA ARQUITECTURA IIAFICHE EL MANIERISMO HISTORIA DE LA ARQUITECTURA II
AFICHE EL MANIERISMO HISTORIA DE LA ARQUITECTURA II
 
BIOMETANO SÍ, PERO NO ASÍ. LA NUEVA BURBUJA ENERGÉTICA
BIOMETANO SÍ, PERO NO ASÍ. LA NUEVA BURBUJA ENERGÉTICABIOMETANO SÍ, PERO NO ASÍ. LA NUEVA BURBUJA ENERGÉTICA
BIOMETANO SÍ, PERO NO ASÍ. LA NUEVA BURBUJA ENERGÉTICA
 
Registro Auxiliar - Primaria 2024 (1).pptx
Registro Auxiliar - Primaria  2024 (1).pptxRegistro Auxiliar - Primaria  2024 (1).pptx
Registro Auxiliar - Primaria 2024 (1).pptx
 
INSTRUCCION PREPARATORIA DE TIRO .pptx
INSTRUCCION PREPARATORIA DE TIRO   .pptxINSTRUCCION PREPARATORIA DE TIRO   .pptx
INSTRUCCION PREPARATORIA DE TIRO .pptx
 
Sesión de clase: Fe contra todo pronóstico
Sesión de clase: Fe contra todo pronósticoSesión de clase: Fe contra todo pronóstico
Sesión de clase: Fe contra todo pronóstico
 
Tema 8.- PROTECCION DE LOS SISTEMAS DE INFORMACIÓN.pdf
Tema 8.- PROTECCION DE LOS SISTEMAS DE INFORMACIÓN.pdfTema 8.- PROTECCION DE LOS SISTEMAS DE INFORMACIÓN.pdf
Tema 8.- PROTECCION DE LOS SISTEMAS DE INFORMACIÓN.pdf
 
Caja de herramientas de inteligencia artificial para la academia y la investi...
Caja de herramientas de inteligencia artificial para la academia y la investi...Caja de herramientas de inteligencia artificial para la academia y la investi...
Caja de herramientas de inteligencia artificial para la academia y la investi...
 
SELECCIÓN DE LA MUESTRA Y MUESTREO EN INVESTIGACIÓN CUALITATIVA.pdf
SELECCIÓN DE LA MUESTRA Y MUESTREO EN INVESTIGACIÓN CUALITATIVA.pdfSELECCIÓN DE LA MUESTRA Y MUESTREO EN INVESTIGACIÓN CUALITATIVA.pdf
SELECCIÓN DE LA MUESTRA Y MUESTREO EN INVESTIGACIÓN CUALITATIVA.pdf
 
PIAR v 015. 2024 Plan Individual de ajustes razonables
PIAR v 015. 2024 Plan Individual de ajustes razonablesPIAR v 015. 2024 Plan Individual de ajustes razonables
PIAR v 015. 2024 Plan Individual de ajustes razonables
 
Valoración Crítica de EEEM Feco2023 FFUCV
Valoración Crítica de EEEM Feco2023 FFUCVValoración Crítica de EEEM Feco2023 FFUCV
Valoración Crítica de EEEM Feco2023 FFUCV
 
ACERTIJO DE POSICIÓN DE CORREDORES EN LA OLIMPIADA. Por JAVIER SOLIS NOYOLA
ACERTIJO DE POSICIÓN DE CORREDORES EN LA OLIMPIADA. Por JAVIER SOLIS NOYOLAACERTIJO DE POSICIÓN DE CORREDORES EN LA OLIMPIADA. Por JAVIER SOLIS NOYOLA
ACERTIJO DE POSICIÓN DE CORREDORES EN LA OLIMPIADA. Por JAVIER SOLIS NOYOLA
 
Fe contra todo pronóstico. La fe es confianza.
Fe contra todo pronóstico. La fe es confianza.Fe contra todo pronóstico. La fe es confianza.
Fe contra todo pronóstico. La fe es confianza.
 
Plan Refuerzo Escolar 2024 para estudiantes con necesidades de Aprendizaje en...
Plan Refuerzo Escolar 2024 para estudiantes con necesidades de Aprendizaje en...Plan Refuerzo Escolar 2024 para estudiantes con necesidades de Aprendizaje en...
Plan Refuerzo Escolar 2024 para estudiantes con necesidades de Aprendizaje en...
 
MAYO 1 PROYECTO día de la madre el amor más grande
MAYO 1 PROYECTO día de la madre el amor más grandeMAYO 1 PROYECTO día de la madre el amor más grande
MAYO 1 PROYECTO día de la madre el amor más grande
 
Presentacion Metodología de Enseñanza Multigrado
Presentacion Metodología de Enseñanza MultigradoPresentacion Metodología de Enseñanza Multigrado
Presentacion Metodología de Enseñanza Multigrado
 
Qué es la Inteligencia artificial generativa
Qué es la Inteligencia artificial generativaQué es la Inteligencia artificial generativa
Qué es la Inteligencia artificial generativa
 
Power Point: Fe contra todo pronóstico.pptx
Power Point: Fe contra todo pronóstico.pptxPower Point: Fe contra todo pronóstico.pptx
Power Point: Fe contra todo pronóstico.pptx
 
Programacion Anual Matemática4 MPG 2024 Ccesa007.pdf
Programacion Anual Matemática4    MPG 2024  Ccesa007.pdfProgramacion Anual Matemática4    MPG 2024  Ccesa007.pdf
Programacion Anual Matemática4 MPG 2024 Ccesa007.pdf
 
ACUERDO MINISTERIAL 078-ORGANISMOS ESCOLARES..pptx
ACUERDO MINISTERIAL 078-ORGANISMOS ESCOLARES..pptxACUERDO MINISTERIAL 078-ORGANISMOS ESCOLARES..pptx
ACUERDO MINISTERIAL 078-ORGANISMOS ESCOLARES..pptx
 

LENGUA Y LITERATURA (1).pdf

  • 1. pregunta:1 Complete el enunciado. En un acto de comunicación, el es el conjunto de señales, signos o símbolos que son objeto de dicho acto. El es el sistema de signos que se utiliza para dar a conocer información que circula por un conducto físico llamado . Opciones: 1 mensaje - código - canal 2 código - mensaje - canal 3 canal - código - mensaje 4 mensaje - canal - código Correcta la opción:1 mensaje - código - canal pregunta:2 Con base en el caso, relacione el elemento de la comunicación con el objeto que lo ejemplifica. Un biólogo lee en su laboratorio una traducción al español del libro Fauna mundial, que trata sobre las ranas marsupiales. Opciones: 1 1a, 2b, 3c 2 1b, 2c, 3a 3 1c, 2d, 3b 4 1d, 2c, 3a Elemento Objeto 1. Código a) Español 2. Canal b) Libro Fauna mundial 3. Contexto c) Laboratorio d) Ranas marsupiales Correcta la opción:1 1a, 2b, 3c pregunta:3 Con base en el texto, relacione el elemento de la comunicación con su componente.
  • 2. El adivino Instalado en la plaza pública, un adivino se entregaba a su oficio. De repente se le acercó un vecino, anunciándole que las puertas de su casa estaban abiertas y que habían robado todo lo que había en su interior. El adivino levantose de un salto y salió corriendo hacia su casa, desencajado y suspirando, para ver lo que había sucedido. El vecino ledijo: —Oye, amigo, tú que te vanaglorias de prever lo que les ocurrirá a los otros, ¿por qué no has previsto lo que te sucedería a ti? El adivino no supo qué responder […]. Modificado con fines pedagógicos. El adivino. Recuperado el 25 de enero de 2016 en http://bit.ly/1lLqXWN Elemento Componente 1. Emisor a) Lenguaje común para el emisor y elreceptor 2. Receptor b) Las puertas de la casa estaban abiertas y se habían robado todo lo que había en su interior 3. Mensaje c) El adivino 4. Código d) Unvecino Opciones: 1 1a, 2c, 3d, 4b 2 1c, 2b, 3a, 4d 3 1d, 2a, 3b, 4c 4 1d, 2c, 3b, 4a Correcta la opción:4 1d, 2c, 3b, 4a pregunta:4 Con base en el caso, identifique el elemento de la comunicación que varía. Stefhy White, docente de profesión e inglesa de nacimiento, ingresa al aula a paso firme. Haciendo gala de su fuerte y firme voz, saluda en perfecto español a sus estudiantes, acto seguido apoya su bolso en el escritorio y busca algo en su portafolio; encuentra una tiza blanca, se acerca a la pizarra y escribe: Por favor, silencio. Opciones: 1 Emisor 2 Receptor 3 Código 4 Canal
  • 3. Correcta la opción:4 Canal pregunta:5 Identifique el elemento de la comunicación que varía en el caso. Leticia era una mujer que le apasionaba hacer ejercicio semanalmente. Cierta ocasión contrajo una fuerte gripe que le impidió realizar su acostumbrada rutina. Daniela, su vecina, le aconsejó que para curarse rápidamente se tenía que hacer una infusión de leche con ajo y tomárselo en las mañanas. Luego, su madre le recomendó una infusión de tilo. Finalmente, su instructor del gimnasio le sugirió tomarse una naranjada caliente con canela y miel de abeja antes de dormir. Opciones: 1 Receptor 2 Mensaje 3 Canal 4 Emisor Correctalaopción:4 Emisor pregunta:6 Relacione la función comunicativa con su característica. Opciones: 1 1a, 2b, 3c 2 1b, 2a, 3c 3 1c, 2a, 3b 4 1c, 2b, 3a Función Característica 1. Fática a) Influye en el comportamiento del interlocutor 2. Apelativa b) Toma como referencia al propio lenguaje 3. Metalingüística c) Inicia, prolonga, reanuda o finaliza la comunicación Correcta la opción:3 1c, 2a, 3b pregunta:7 Identifique la función del lenguaje en el texto.
  • 4. Ahora que se ha comprobado que la harina de pescado es apta para el consumo humano, los científicos del Gobierno están buscando recetas apetitosas para sacar provecho de sus cualidades de alto contenido de proteínas. Uno de los candidatos es un líquido, la emulsión para bebés, elaborado a base de concentrado de pescado, con el propósito de combatir la desnutrición en los infantes. Camba, E. (2006). Opciones: 1 Emotiva 2 Referencial 3 Poética 4 Metalingüística Correcta la opción:2 Referencial pregunta:8 Seleccione las funciones del lenguaje presentes en el texto. El saxofonista Lester Young, fue el primero en decir “I am cool”, refiriéndose a que cool significa “estar relajado en un determinado ambiente, con la situación bajo control" […] Cool es una suerte de sublimación de aquello que jamás ha sido aceptado. Ponerle marca a lo no aceptado, a lo freak, a lo mínimo, a lo “pasado de moda”. Ponerle un sello a la diferencia, y paradójicamente, normalizarla, banalizarla. Quitarle su belleza original y venderla. Los empresarios sospechan que en la minoría hay una fuerza latente peligrosa, y antes de que explote y de que sea una voz, la compran. Después la venden. Y nosotros la compramos […] Franco, A. Rebeldes que no se despeinan. Revista Mundo Diners. Quito. Recuperado el 18 de julio de 2014 en http://www.revistamundodiners.com/?p=4077 1. Apelativa 2. Emotiva 3. Informativa 4. Fática 5. Metalingüística Opciones: 1 1, 2, 3 2 1, 3, 4 3 1, 3, 5 4 2, 4, 5 Correcta la opción:3
  • 5. 4 1, 3, 5 pregunta:9 Identifique la función del lenguaje. […] A todas horas de la noche, en las fiestas patrias, en el aniversario del descubrimiento de América, dispuesto a solidarizarme con lo que sea, víctima de mi solidaridad. Inútil, completamente inútil, que me resista. La solidaridad ya es un reflejo en mí, algo tan inconsciente como la dilatación de las pupilas. Si durante un centésimo de segundo consigo desolidarizarme de mi solidaridad, en el centésimo de segundo que lo sucede, sufro un verdadero vértigo de solidaridad. Solidario de las olas sin velas... sin esperanza. Solidario del naufragio de las señoras ballenatos, de los tiburones vestidos de frac, que les devoran el vientre y la cartera. Solidario de las carteras, de los ballenatos y de los fraques […]. Girondo, O. (s/f). Se podrá discutir mi erudición ornitológica y la eficacia de mis aperturas de ajedrez. Recuperado el 19 de noviembre de 2015 en http://bit.ly/1O6EiEM Opciones: 1 Fática 2 Conativa 3 Estética 4 Metalingüística Correcta la opción:3 Estética pregunta:10 Con base en el texto, identifique el nivel de lenguaje. La primera vez que se me ocurrió tentar la experiencia a cuyo relato están dedicadas estas líneas, fue una tarde, leyendo no sé dónde, que los naturales de Java atribuían la falta de lenguaje articulado en los monos a la abstención, no a la incapacidad. No hablan, decían, para que no los hagan trabajar. Semejante idea, nada profunda al principio, acabó por preocuparme hasta convertirse en este postulado antropológico: Los monos fueron hombres que por una u otra razón dejaron de hablar. El hecho produjo la atrofia de sus órganos de fonación y de los centros cerebrales del lenguaje; debilitó, casi hasta suprimirla, la relación entre unos y otros, fijando el idioma de la especie en el grito inarticulado, y el humano primitivo descendió a ser animal. Lugones, L. Izur. Recuperado el 10 de mayo de 2015 en http://www.ciudadseva.com/textos/cuentos/esp/lugones/yzur.htm Opciones: 1 Coloquial 2 Culto 3 Vulgar
  • 6. 3 Familiar Correcta la opción:2 Culto pregunta:11 Identifique el nivel de lenguaje empleado en el texto. Cierta noche, al regresar a casa, hallé, fijada a la puerta, una breve nota en que mi hija de 14 años de edad me decía: “Adorada mamá, echa un vistazo al horno”. Allí encontré un segundo billete: “Está muy limpio, ¿no crees? Y ahora, fíjate en el piso de la cocina”. Otras notas parecidas me fueron guiando por toda la casa, recién aseada y puesta en orden. Sobre mi cama, un último papelito rezaba: “Te adoro, mamá”. B. J. B. (1978). Selecciones del Reader’s Digest. Opciones: 1 Culto 2 Vulgar 3 Coloquial 4 Científico Correcta la opción:3 Coloquial pregunta:12 Con base en el texto, seleccione los extranjerismos. Iván era un músico talentoso que había perfeccionado su técnica como ninguno a la hora de interpretar la balalaica. Su carisma lo había llevado a grandes escenarios alrededor del mundo y a recibir decenas de distinciones como reconocimiento a su talento. Cuando era niño soñó con pelear en la guerra, pero una noche en una reunión familiar, Iván vio que un jovencito que atendía el samovar desenfundó el curioso instrumento musical y empezó a sacarle las más tristes melodías. El sonido impactó al niño y abandonó sus sueños bélicos para dedicarse a revivir la pasión de su pueblo por el olvidado instrumento. 1. Balalaica 2. Escenarios 3. Distinciones 4. Guerra 5. Samovar 6. Bélicos Opciones: 1 1, 2, 6 2 1, 4, 5
  • 7. 2, 3, 4 4 3, 5, 6 Correctalaopción:2 1, 4, 5 pregunta:13 Relacione la oración con el nivel de lenguaje utilizado. Nivel Oración Opciones: 1 1b, 2a, 3c, 4d 2 1b, 2c, 3a, 4d 3 1d, 2c, 3a, 4b 4 1d, 2c, 3b, 4a 1. Vulgar a) Este es un hábitat de fauna endémica 2. Formal b) Mi hijo trajo a sus guaguas 3. Científico c) Tenga la gentileza de sentarse 4. Familiar d) Es la misma señora que vino denantes Correcta la opción:3 1d, 2c, 3a, 4b pregunta:14 En un periódico local se han impreso varios titulares, identifique el que usa extranjerismos. Opciones: 1 Triunfan los llapingachos en festival de comida internacional 2 Cambio en hipertexto afectó a los mensajes en las principales redes 3 Gobierno entregará ordenadores a estudiantes de comunidades rurales 4 Vecinos denuncian un parking improvisado junto a un terreno Correcta la opción:4 Vecinos denuncian un parking improvisado junto a un terreno pregunta:15 Seleccione los enunciados que contienen extranjerismos.
  • 8. 1. Las palabras pronunciadas por el autor parecían adecuadas al cliché 2. En el mundo del deporte, aquel joven tenista es considerado como un monstruo 3. Me encanta cenar en este restaurante porque su cocinero es todo un gourmet 4. Los actores llegaron temprano para ensayar los diálogos y cumplir surol 5. Tenía muy irritados los ojos y el exceso de polución le causabaprurito 6. Después de aprobar todos los cursos, empezarán el periodo de praxis Opciones: 1 1, 2, 5 2 1, 3, 4 3 2, 4, 6 4 3, 5, 6 Correctalaopción:2 1, 3, 4 pregunta:16 Elija las palabras que se forman con prefijos. 1. Medidor 2. Primavera 3. Inconveniente 4. Inmortalizar 5. Kilómetros 6. Penúltimo Opciones: 1 1, 2, 3, 4 2 1, 2, 4, 5 3 2, 3, 5, 6 4 3, 4, 5, 6 Correctalaopción:4 3, 4, 5, 6 pregunta:17 Seleccione las palabras formadas con prefijos. 1. Destellante 2. Franquicia 3. Desmesura 4. Tetracordio 5. Posadeño
  • 9. 6. Posponer Opciones: 1 1, 2, 5 2 1, 3, 6 3 2, 4, 5 4 3, 4, 6 Correctalaopción:4 3, 4, 6 pregunta:18 Seleccione las palabras formadas con prefijos. 1. Abadía 2. Anormalidad 3. Acuático 4. Adyacente 5. Antropomorfo 6. Anacrónico Opciones: 1 1, 3, 5 2 1, 4, 6 3 2, 3, 5 4 2, 4, 6 Correctalaopción:4 2, 4, 6 pregunta:19 Seleccione las palabras formadas con sufijos. 1. Brillantez 2. Producto 3. Sacudida 4. Egoísmo 5. Policéfalo 6. Torno Opciones: 1 1, 2, 6
  • 10. 2 1, 3, 4 3 2, 4, 5 4 3, 5, 6 Correctalaopción:2 1, 3, 4 pregunta:20 Seleccione las palabras formadas por el prefijo que signifca negación. 1. Descarga 2. Amorfo 3. Incógnito 4. Impecable 5. Anarquista 6. Sinfonía Opciones: 1 1, 2, 3, 4 2 1, 3, 5, 6 3 2, 3, 4, 5 4 2, 4, 5, 6 Correctalaopción:3 2, 3, 4, 5 pregunta:21 Seleccione las palabras que se componen con sufijos despectivos. 1. Caserío 2. Aldehuela 3. Viejísima 4. Libraco 5. Policastro 6. Peineta Opciones: 1 1, 3, 6 2 2, 3, 5 3 2, 4, 5 4
  • 11. 3, 4, 6 Correctalaopción:3 2, 4, 5 pregunta:22 Con base en el texto, identifique la palabra formada con el prefijo que significa privación. En sus páginas interminables hay mapas de lugares trastocados por la memoria, y planos de ciudades que se han convertido en silencio; los recuerdos de un hombre que caminó toda la vida alrededor de su cama hasta sentir que descubrió el Polo Norte y la larga canción sin música de un flautista demencial que jura poder hablar con las cigüeñas. Modificado con fines pedagógicos. Recuperado el 3 de marzo de 2017 en http://bit.ly/2mUXTO3 Opciones: 1 Trastocados 2 Interminables 3 Flautista 4 Recuerdos Correcta la opción:2 Interminables pregunta:23 Identifique la palabra compuesta por el prefijo que significa dificultad o anomalía. Opciones: 1 Sinfonía 2 Perifonía 3 Disfonía 4 Afonía Correcta la opción:3 Disfonía pregunta:24 Identifique la palabra compuesta por el prefijo que significa a la vez. Opciones: 1 Subconjunto 2 Analgesia
  • 12. 3 Sincronía 4 Adverbio Correcta la opción:3 Sincronía pregunta:25 Identifique el tiempo en el que se encuentra conjugado el verbo en la oración. Todas las tardes venía por el pan. Opciones: 1 Pretérito perfecto compuesto 2 Pretérito perfectosimple 3 Pretérito pluscuamperfecto 4 Pretérito imperfecto Correcta la opción:4 Pretérito imperfecto pregunta:26 Identifique el tiempo del verbo resaltado en negrita. Él leía novelas cortas para entretenerse. Opciones: 1 Presentesubjuntivo 2 Pretéritoimperfecto 3 Pretéritoindefinido 4 Potencial simple Correcta la opción:2 Pretérito imperfecto pregunta:27 Identifique la forma verbal conjugada en pretérito perfecto. La víspera, en la catedral se celebraba una solemne ceremonia fúnebre que impidió que se hicieran preparativos de ninguna clase. Como consecuencia, en una sola mañana había que vestir con colgaduras de damasco rojo todas las columnas góticas de las tres naves, que tenían unos treinta pies de elevación. El obispo había mandado venir de París cuatro tapiceros; pero esos señores no lograban hacerlo todo, y por añadidura, tampoco tenían grandes ganas de trabajar.
  • 13. Modificado con fines pedagógicos. Stendhal. (s/f). Rojo y negro. Recuperado el 19 de noviembre de 2015 en http://bit.ly/SZqXVp Opciones: 1 Celebraba 2 Tenían 3 Impidió 4 Lograban Correcta la opción:3 Impidió pregunta:28 Seleccione las partes variables de la oración. La casa de mi abuelo tiene un jardín muy amplio. 1. La 2. De 3. Abuelo 4. Un 5. Muy Opciones: 1 1, 2, 3 2 1, 3, 4 3 2, 3, 5 4 2, 4, 5 Correctalaopción:2 1, 3, 4 pregunta:29 Identifique las partes invariables de la oración. En el zoológico hay algunos monos, osos, tortugas, llamas, tigrillos y un cóndor. Opciones: 1 zoológico - un 2 el - hay
  • 14. 3 algunos - cóndor 4 en - y Correcta la opción:4 en - y pregunta:30 Con base en el texto, elija las palabras variables. Lo que aquel hombre asimiló fue que el capullo restrictivo y la lucha eran necesarias para que la mariposa pudiera salir por el diminuto agujero, era la manera que utilizaba la naturaleza para enviar fluido del cuerpo de la mariposa hacia sus alas, de modo que estuviera lista para volar tan pronto obtuviere la libertad del capullo. Todo esfuerzo es vital para capturar nuestras metas. Modificado con fines pedagógicos. Recuperado el 06 de septiembre de 2017 en http://www.doslourdes.net/la_lecci%C3%B3n_de_la_mariposa.htm 1. Que 2. Restrictivo 3. De modo que 4. Asimiló 5. Para que 6. Obtuviere Opciones: 1 1, 4, 5 2 1, 5, 6 3 2, 3, 5 4 2, 4, 6 Correctalaopción:4 2, 4, 6 pregunta:31 Identifique una parte invariable de la oración. Mas no pasó mucho tiempo para que el águila recibiera el pago de su traición. Opciones: 1 Mas 2 Pasó 3 Su 4
  • 15. El Correcta la opción:1 Mas pregunta:32 Complete el enunciado. En la oración "El perro anciano tiene ahora una expresión de ensueño"; las palabras y son invariables porque carecen de inflexiones. Opciones: 1 perro -anciano 2 el - una 3 expresión -ensueño 4 ahora - de Correcta la opción:4 ahora - de pregunta:33 Elija las partes variables de la oración. Luis o Karla nos contarán todo cuando regresen. 1. Luis 2. O 3. Nos 4. Contarán 5. Cuando 6. Regresen Opciones: 1 1, 2, 3, 5 2 1, 3, 4, 6 3 2, 4, 5, 6 4 3, 4, 5, 6 Correctalaopción:2 1, 3, 4, 6 pregunta:34 Seleccione las partes variables de la oración.
  • 16. La poesía es un territorio que salva de la intemperie. 1. Es 2. Un 3. Que 4. De 5. La Opciones: 1 1, 2, 5 2 1, 3, 4 3 2, 3, 5 4 3, 4, 5 Correctalaopción:1 1, 2, 5 pregunta:35 Identifique las conjunciones. 1. Sino 2. En 3. Y 4. Que 5. Hasta 6. Un 7. Pero Opciones: 1 1, 2, 5, 7 2 1, 3, 4, 7 3 2, 3, 4, 6 4 3, 5, 6, 7 Correctalaopción:2 1, 3, 4, 7 pregunta:36 Identifique la función sintáctica del grupo nominal en negrita. El vocablo yo-yo o yoyó, pues de las dos formas se puede escribir, procede del tagalo, la lengua nativa de Filipinas, y significa volver. Hasta hace 400 años, los filipinos usaban el yoyó como arma. No obstante, el origen de este juguete parece estar en China, aunque también era conocido en Grecia por lo menos hace 2 500 años.
  • 17. Los artesanos helenos lo fabricaban con madera, metal o terracota, y decoraban cada pieza con dibujos de los dioses. Algunos de estos primeros yoyós pueden contemplarse en el Museo Metropolitano de Arte, en Nueva York. Muy Interesante. ¿Quién inventó el yoyó? Recuperado el 27 de enero de 2016 en bit.ly/23t9Qen Opciones: 1 Objeto indirecto 2 Complemento circunstancial 3 Objeto directo 4 Núcleo del sintagma verbal Correcta la opción:2 Complemento circunstancial pregunta:37 Identifique la función sintáctica del sintagma en negrita. Se convirtieron en mi mayor tesoro los viejos libros de la biblioteca. Opciones: 1 Aposición 2 Núcleo del sujeto 3 Modificadordirecto 4 Modificador indirecto Correcta la opción:4 Modificador indirecto pregunta:38 Relacione el elemento sintáctico con su definición. Elemento Definición 1. Complemento indirecto a) Determinante del núcleo del predicado que señala alguna circunstancia particular en que ocurre la acción del verbo 2. Núcleo del sujeto b) Construcción sintáctica formada por un término unido al núcleo del sujeto por medio de un nexo, su función es explicativa 3. Complemento circunstancial Opciones: 1 c) Sustantivo o pronombre que realiza la acción del predicado
  • 18. 1a, 2b, 3c 2 1b, 2c, 3a 3 1c, 2a, 3b 4 1c, 2b, 3a Correcta la opción:2 1b, 2c, 3a pregunta:39 Identifique la definición del objeto indirecto. Opciones: 1 Parte de la oración que recibe de forma directa la acción del verbo y se reconoce porque se sustituye con los pronombres personales átonos "lo", "la", "los" o "las" 2 Elemento sintáctico que acompaña al núcleo del sujeto de manera directa en forma de artículo o adjetivo y concuerda con él en género y número 3 Parte de la oración compuesta por un conjunto de palabras que se añaden al núcleo del sujeto a través de preposiciones 4 Elemento sintáctico que evidencia al receptor de los efectos de la acción verbal, va precedido por una de las preposiciones "a" o "para", y puede ser reemplazado por los pronombres "le" o "les" Correcta la opción:4 Elemento sintáctico que evidencia al receptor de los efectos de la acción verbal, va precedido por una de las preposiciones "a" o "para", y puede ser reemplazado por los pronombres "le" o "les" pregunta:40 Con base en el texto, identifique la función sintáctica de los términos en negrita. El vocablo yo-yo o yoyó, pues de las dos formas se puede escribir, procede del tagalo, la lengua nativa de Filipinas, y significa volver. Hasta hace 400 años, los filipinos usaban el yoyó como arma. No obstante, el origen de este juguete parece estar en China, aunque también era conocido en Grecia por lo menos hace 2 500 años. Los artesanos helenos lo fabricaban con madera, metal o terracota y decoraban cada pieza con dibujos de los dioses. Algunos de estos primeros yoyós pueden contemplarse en el Museo Metropolitano de Arte, enNueva York. Muy Interesante. ¿Quién inventó el yoyó? Recuperado el 27 de enero de 2016 en bit.ly/23t9Qen Opciones: 1 Modificadordirecto 2 Aposición 3 Modificador indirecto
  • 19. 4 Núcleo del sintagma nominal Correcta la opción:3 Modificador indirecto pregunta:41 Identifique la función del sintagma en negrita. El episodio final de la serie, cual primicia noticiosa, será visto por millones de espectadores. Opciones: 1 Predicativo 2 Agente 3 Complemento circunstancial 4 Complemento indirecto Correcta la opción:2 Agente pregunta:42 Identifique el objeto directo en la oración. Las docentes de Guayaquil escriben poesías para sus estudiantes. Opciones: 1 Escriben 2 Docentes 3 Poesías 4 Guayaquil Correcta la opción:3 Poesías pregunta:43 Identifique la función del sintagma en negrita. La joven doctora, Mariana Acosta, es una investigadora mundialmente conocida por sus descubrimientos genéticos. Opciones: 1 Complemento directo 2
  • 20. Complementoindirecto 3 Agente 4 Atributo Correctalaopción:4 Atributo pregunta:44 Relacione el vicio de dicción con la oración que lo ejemplifica. Opciones: 1 1a, 2c, 3e, 4b, 5d 2 1b, 2e, 3d, 4a, 5c 3 1c, 2e, 3a, 4d, 5b 4 1d, 2a, 3c, 4e, 5b Vicio Oración 1. Cacofonía a) Salga afuera a ver si llegó el estudiante 2. Pleonasmo b) Clickea en tu mouse a ver si funciona 3. Muletilla c) Este, dime de nuevo cómo pasó 4. Solecismo d) Paúl, para para pensar 5. Neologismo e) Bajo el punto de vista del señor, se podría decir que es así Correcta la opción:4 1d, 2a, 3c, 4e, 5b pregunta:45 Seleccione las oraciones en las que se ha incurrido en dequeísmo. 1. Me enteré de que llegarías después de las vacaciones de invierno 2. Los últimos en llegar nos dijeron de que todo estaba muy bien 3. Los empleadores creyeron que la cuenta estaba cerrada desde ayer 4. Insistieron de que tratáramos de venir pronto Opciones: 1 1, 3 2 1, 4 3 2, 3 4 2, 4
  • 21. 2, 4 pregunta:46 Relacione el vicio de dicción con su ejemplo. Vicio Ejemplo 1. Cacofonía a) Me enteré que esa relación de años se ha rompido por nimiedades 2. Conjugación verbal incorrecta b) La agua oxigenada tiene propiedades cicatrizantes 3. Neologismo c) Debes ser proactiva en tutrabajo d) Sé que en ese cubículo cabo sin problema e) Yo lo coloco y ella lo quita f) Solo debes clickear en el enlace de la red Opciones: 1 1ab, 2df, 3ce 2 1bc, 2ad, 3ef 3 1be, 2ad, 3cf 4 1ef, 2bc, 3ad Correcta la opción:3 1be, 2ad, 3cf pregunta:47 Relacione el vicio de construcción con la oración que lo ejemplifica. Vicio Oración 1. Anfibología a) La situación se puso intransigente 2. Dequeísmo b) Contó que cuando ya no aguantó más, salió afuera a llorar 3. Redundancia c) Le dijo de que volviera mañana en la tarde 4. Uso incorrecto de acuerdo con el significado d) No puedo accesar al programa para printear el documento 5. Barbarismo e) Salió tan impresionada de la película que temblaba Opciones: 1 1b, 2c, 3e, 4d, 5a 2 1c, 2b, 3d, 4e, 5a 3 1d, 2e, 3a, 4b, 5c 4 1e, 2c, 3b, 4a, 5d
  • 22. 1e, 2c, 3b, 4a, 5d pregunta:48 Identifique la oración que presenta un vicio de construcción. Opciones: 1 Debes de obedecer a tu papá 2 Asistimos al concierto de salsa 3 Trabajo tanto que me siento estresada 4 Los estudiantes deben desarrollar habilidades lingüísticas Correcta la opción:1 Debes de obedecer a tu papá pregunta:49 Identifique el enunciado que presenta un vicio de palabra o cláusula a la que puede darse más de una interpretación. Opciones: 1 Le llamó para confirmar que sería contratado 2 Estamos convencidos que ganaremos el juego 3 Me informó de que no era necesario reportarlos 4 Se arrienda habitación para estudiantes con cama Correcta la opción:4 Se arrienda habitación para estudiantes con cama pregunta:50 Identifique el enunciado que presenta un caso de queísmo. Opciones: 1 ¡Qué emoción! El próximo fin de semana regresan mis familiares 2 Me avisaron de que mañana saldremos a caminar por la carretera 3 Nunca cayó en cuenta que había olvidado las llaves de su casa antes de salir 4 Todavía no entiendo sus decisiones, ¿qué razones aparentaban lo contrario? Correcta la opción:3 Nunca cayó en cuenta que había olvidado las llaves de su casa antes de salir pregunta:51
  • 23. 1, 3, 5 Identifique la oración en la que la coma cumple la función de limitar una aclaración o ampliación. Opciones: 1 José tenía como juguetes un balón, un carro, una patineta y una bicicleta 2 Los jugadores saltaron al campo de juego, el público enloquecía y las barras no dejaban de alentar 3 Descartes, notable matemático francés, hizo muchos aportes en sus escritos 4 Juan, ven por favor Correcta la opción:3 Descartes, notable matemático francés, hizo muchos aportes en sus escritos pregunta:52 Seleccione las oraciones que utilizan de forma correcta los signos de puntuación. 1. La bandera ecuatoriana es amarilla azul y roja. 2. Guayaquil, puerto principal del Ecuador, es famosa por su gastronomía. 3. Mi hijo mayor practica baloncesto, el segundo, fútbol, y el tercero, tenis. 4. Tania no preparó los exámenes; sin embargo, pasó al siguiente curso. 5. Tomás no paraba de llorar; por consiguiente, fue hospitalizado. Opciones: 1 1, 2, 4 2 1, 3, 5 3 2, 3, 4 4 2, 4, 5 Correctalaopción:4 2, 4, 5 pregunta:53 Seleccione las oraciones que utilizan correctamente las normas de puntuación. 1. ¿Se le ofrece café, té, o agua? 2. La región Amazónica es extensa, exuberante y calurosa. 3. Las puertas, ventanas, y armarios quedaron destruídos. 4. Él, que no era tonto, se dio cuenta al instante. 5. Cortaron los árboles, limpiaron el jardín, y quedaron exhaustos. Opciones: 1 1, 2, 4 2
  • 24. 2, 4, 5 3 2, 3, 5 4 2, 4, 5 Correctalaopción:4 2, 4, 5 pregunta:54 Seleccione las oraciones en las que falta colocar punto y coma (;). 1. El bebé tomó su cena, jugó un rato más y se quedó dormido en brazos de su madre. 2. Mi madre trajo dulces, empanadas y refrescos en seguida se puso a preparar la comida. 3. Llovió toda la noche. Al otro día los campos amanecieron inundados y empantanados. 4. Los síntomas de la gripe empeoraron: tos, catarro, fiebre para colmo me dolía la cabeza. 5. Parecía muy contento con el trabajo, la compañía y todo pero al otro mes estaba agotado. Opciones: 1 1, 2, 4 2 1, 3, 4 3 2, 3, 5 4 2, 4, 5 Correctalaopción:4 2, 4, 5 pregunta:55 Elija los enunciados en los que se utilizan correctamente las normas de acentuación. 1. La niña que esta frente a Juan es una de las más inteligentes del salón 2. ¡Esta no es vida! Gritó el pescador y se echó a la mar mientras silbaba la tonada 3. No era necesaría tanta explicación para entender el concepto galo detruhan 4. Ayer, todo estuvo listo para la grabación: las luces, el guion, las cámaras, el set… 5. Cuando me habló de estudiar Astronomía, creí que era una de sus tantasbromas Opciones: 1 1, 2, 4 2 1, 3, 5 3 2, 3, 4 4 2, 4, 5 Correcta la opción:4
  • 25. pregunta:56 Elija las oraciones que respetan las reglas de acentuación. 1. El perro de Juan siempre juega con él 2. María se fue a trabajar, eso yo lo se 3. Quería hacer más ejercicio, mas tuve que volver a mi casa 4. Mi auto está pintado de un color pedido especialmente por mí 5. Si yo dijera siempre que si, estaría perdido 6. De vez en cuando, las personas necesitan que se les dé un consejo Opciones: 1 1, 2, 3, 5 2 1, 3, 4, 6 3 2, 3, 4, 5 4 2, 4, 5, 6 Correctalaopción:2 1, 3, 4, 6 pregunta:57 Elija las oraciones en las que se emplean de forma adecuada las mayúsculas. 1. Me dijeron que María está haciendo compras en el mercado con luisa 2. Juan y Pedro son maestros de lengua y literatura 3. La federación de fútbol de Brasil inaugura el mundial en junio 4. Los estudiantes de séptimo año de educación básica fueron de paseo aLoja 5. El hospital doce de octubre es el mejor de la ciudad 6. El Viernes Santo es un día de descanso obligatorio para los empleadospúblicos Opciones: 1 1, 3, 4 2 2, 3, 5 3 2, 4, 6 4 4, 5, 6 Correctalaopción:3 2, 4, 6 pregunta:58 Identifique la oración en la que se usan correctamente las mayúsculas. Opciones: 1
  • 26. La brújula señala el Norte 2 El Norte de la ciudad está inundado 3 Viajamos por el Norte de Ecuador 4 El viento del Norte es más suave Correcta la opción:1 La brújula señala el Norte pregunta:59 Identifique el enunciado que contiene el conector que expresa contraste. Opciones: 1 Sus amigos guardaron silencio, a la vez, su familia aplaudió complacida 2 Sus amigos guardaron silencio, asimismo, su familia aplaudió complacida 3 Sus amigos guardaron silencio, en cambio, su familia aplaudió complacida 4 Sus amigos guardaron silencio, por último, su familia aplaudió complacida Correcta la opción:3 Sus amigos guardaron silencio, en cambio, su familia aplaudió complacida pregunta:60 Identifique el tipo de conector que se utiliza en la oración. Después de la comida, tomaron un café en el salón. Opciones: 1 Causal 2 Adversativo 3 Comparativo 4 Temporal Correcta la opción:4 Temporal pregunta:61 Relacione la palabra con la oración en la que se encuentra su antónimo. Palabra Oración 1. Tácito a) El gato de la casa es muy arisco 2. Amigable b) Aquel hombre la trataba con descortesía
  • 27. 3. Amabilidad c) Lo que dice no está expreso Opciones: 1 1a, 2c, 3b 2 1b, 2a, 3c 3 1c, 2a, 3b 4 1c, 2b, 3a Correcta la opción:3 1c, 2a, 3b pregunta:62 Identifique el antónimo de la palabra en negrita. […] Con todos aquellos personajes de la aristocracia parisina fue tejiendo otro mundo literario en una evocación magistral que, más que copiarla según el realismo tradicional, lo evoca con la sutileza de un pintor impresionista. De hecho, la novela de Proust se ha comparado con la pintura de Monet y la música de Debussy por su pretensión de captar la luz y el instante, y de extraer de la realidad la impresión fugaz de la misma […]. Ponte Far, J. (2015). Marcel Proust, uno de los grandes. Recuperado el 16 de noviembre de 2015 en http://bit.ly/1RZHpxw Opciones: 1 Duradera 2 Efímera 3 Transitoria 4 Perecedera Correcta la opción:1 Duradera pregunta:63 Seleccione las oraciones que contienen palabras homógrafas. 1. ¡Que se alce el sol y brille! -dijo el granjero aquella mañana. En el prado divisó un alce que se alimentaba del pasto 2. Ese instante, una hermosa mujer abría la puerta y se acercaba hacia él... habría de ser un día feliz 3. Su esposa había regresado, podrían seguir juntos su camino. "Hoy camino contigo" le dijo ella 4. A pesar de coser la ropa y de cocer la fruta para las mermeladas, la mujer mantenía su buen humor 5. Gracias a ella, la llama de su relación seguía. Su nombre reflejaba su carácter, "se llama Alegría" Opciones: 1 1, 2, 5
  • 28. 2 1, 3, 4 3 1, 3, 5 4 2, 3, 4 Correctalaopción:3 1, 3, 5 pregunta:64 Seleccione las palabras homógrafas. 1. Acerbo 2. Prensa 3. Alce 4. Vacilo 5. Vino Opciones: 1 1, 2, 3 2 1, 3, 4 3 2, 3, 5 4 2, 4, 5 Correctalaopción:3 2, 3, 5 pregunta:65 Seleccione las características propias de una composición satírica. 1. Se manifiesta en tono mordaz 2. Presenta temas con los que un grupo humano se identifica 3. Exterioriza los lamentos tras la defunción de un ser querido 4. Expone al ridículo a alguien o algo Opciones: 1 1, 3 2 1, 4 3 2, 3 4 2, 4 Correcta la opción:2
  • 29. 1, 4 pregunta:66 Con base en el texto, identifique el subgénero literario. A un niño de un pueblo le regalaron un pequeño pájaro. Al recibirlo, el niño le ató una pata a un hilo y lo mantuvo agarrado. Luego lo lanzó para que volara, pero el animal se quedó quieto. El niño le dijo: - Animal lento, te doy permiso y espacio para que extiendas tus alas y vueles, pero tú te quedas totalmente quieto. No sabes gozar de tu libertad. El pájaro, molesto, le contesto: - No soy lento, solo soy astuto. Esa libertad que me das es solo apariencia y ese bien pronto se tornará en un mal. Apenas intente volar, tú me jalarás por el hilo atado a mi pata. “No hay peor esclavitud que una en la que se crea que se es libre, cuando no lo es”. El niño y el pájaro. (s.f.). Recuperado el 07 de abril de 2017 en http://bit.ly/2oMJFTT Opciones: 1 Cuento 2 Fábula 3 Leyenda 4 Mito Correcta la opción:2 Fábula pregunta:67 Eleonor debe elaborar un ensayo que justifique que Otelo es una tragedia, por lo que, con el fin de evidenciar características de este subgénero, ha realizado una cita textual de la obra. Con base en el fragmento, identifique el argumento que eligió Eleonor para desarrollar su tarea. EMILIA.- ¡Buenos caballeros, permitidme que hable! Es justo que lo obedezca, pero no ahora. ¡Quizá, Yago, no vuelva nunca al hogar! OTELO.- ¡Oh, oh, oh! (Cae sobre el lecho) EMILIA.-¡Sí! ¡Déjate caer y ruge! ¡Pues ha matado a la más tierna inocente que alzó jamás los ojos al cielo! OTELO.- (Levantándose) ¡Oh! ¡Era impura! (A Graciano) Apenas os conozco, tío. ¡Ahí yace vuestra sobrina, cuyo aliento, en verdad, acaban de cortar estas manos! ¡Sé que este acto aparece horrible y cruel! GRACIANO.- ¡Pobre Desdémona! ¡Cuánto me alegro de que no exista tu padre! ¡Tu casamiento fue para él un golpe mortal, y la sola pena que cortó en dos el viejo hilo de su vida! Si viviera ahora, este espectáculo le impulsaría a algún acto de desesperación. ¡Sí! ¡Maldeciría a su buen ángel, le arrojaría de su lado y se atraería la reprobación del cielo! Shakespeare, W. (s.f.). Otelo: el moro de Venecia. Recuperado el 05 de abril de 2017 en http://bit.ly/1DQCtaV Opciones: 1 Este fragmento denota un ámbito de conflicto en el que se aprecian realidades normales de la vida cotidiana, por lo tanto, corresponde a una tragedia 2
  • 30. Este fragmento muestra la interacción de varios personajes: Emilia, Otelo y Graciano, de forma chabacana y grotesca, por lo tanto, corresponde al subgénero de la tragedia 3 En la obra Otelo: moro de Venecia, la acción dramática desemboca en un mensaje religioso, basado en temas morales o sacros, por ello es una tragedia 4 En la obra Otelo: moro de Venecia, las escenas reflejan situaciones centradas en el sufrimiento y las peripecias dolorosas de la vida humana, por lo tanto, es una tragedia Correcta la opción:4 En la obra Otelo: moro de Venecia, las escenas reflejan situaciones centradas en el sufrimiento y las peripecias dolorosas de la vida humana, por lo tanto, es una tragedia pregunta:68 Con base en el fragmento, identifique el tipo de texto no literario. - Ha escrito usted un libro muy divertido. - Quise hacerlo así. La verdad es que publicar libros es divertido. Para mí hubiera sido muy difícil escribir sobre el mundo editorial sin sentido del humor. - ¿Siempre ha sido así? - Siempre. Cada libro, cada manuscrito que entra por esa puerta es lo más importante en la vida del autor. Hay excepciones entre los libros que se escriben y que se publican, pues hay libros para perder peso, diccionarios, libros para hacer mejor el amor; pero los que me llegan a mí representan, sin excepciones, lo más importante en la vida de sus autores […] Cruz, J. (6 de noviembre de 2005). El editor en el trapecio. El País (archivo digital). Recuperado el 24 de julio de 2014 en http://bit.ly/1rOVAJC Opciones: 1 Noticia 2 Entrevista 3 Editorial 4 Crónica Correcta la opción:2 Entrevista pregunta:69 Identifique el tipo de texto. He aquí uno de los géneros más apasionados e ingratos del periodismo: la entrevista a una estrella de rock. La principal constatación de quienes hemos pasado por el calvario de aguardar a una luminaria en el banquillo de las expectativas es que ningún héroe del alto volumen quiere hablar. Profesionales de la intensidad, quienes expresan adrenalina en un escenario suelen relajarse con otras formas del exceso, de la destrucción de televisores al acopio de pandas de peluches, motocicletas, castillos medievales y otras variantes del coleccionismo extremo. La entrevista es una convención tediosa. Villoro, J. (2016). La entrevista a una estrella de rock. Modificado con fines pedagógicos. Recuperado el 18 de febrero de 2016 en http://bit.ly/1PIxkVJ
  • 31. Opciones: 1 Crónica 2 Artículo deopinión 3 Noticia 4 Reportaje científico Correcta la opción:2 Artículo de opinión pregunta:70 Identifique la clase de texto no literario que se muestra a continuación. Cuatro nuevas especies de ranas terrestres fueron halladas en los bosques del río Upano, provincia de Morona Santiago, en la vertiente oriental de la cordillera de los Andes. Esto, tras un estudio realizado por tres biólogos ecuatorianos que evaluaron ocho localidades de la subcuenca alta del río. Las expediciones les permitieron obtener abundante información sobre las ranas terrestres Pristimantis, con el objetivo de construir una sinopsis del género, identificar especies nuevas y determinar sus patrones de diversidad. El estudio revela que la cuenca del río Upano contiene 37 especies de ranas terrestres. De estas, 25 han sido previamente reportadas en los bosques de los Andes orientales de Ecuador, 16 de ellas son endémicas, nueve amenazadas, ocho son descritas como especies candidatas a nuevas y cuatro son identificadas en el estudio como nuevas. Alarcón, I. (2017). Ecuador cuenta con cuatro nuevas especies de ranas terrestres. Recuperado el 4 de abril de 2017 en http://bit.ly/2nPN2FV Opciones: 1 Editorial 2 Crónica 3 Noticia 4 Reseña Correcta la opción:3 Noticia pregunta:71 Identifique el tipo de texto no literario. Desde hace un tiempo, viene dándose un fenómeno curioso en los segmentos de ciencia que se presentan en varios noticieros televisivos. Cada vez que se hace un alegato en defensa de la ciencia, o se presenta a científicos que reclaman más ayudas, presupuestos o becas, se muestra en pantalla a gente en bata. Personalmente, me preocupa que se genere la sensación de que la única ciencia necesaria es aquella que tiene un impacto inmediato en la sociedad. No pretendo que se reduzca la visibilidad de estos heroicos
  • 32. investigadores, lo único que pretendo es recordar que la investigación científica es necesaria en todos los ámbitos, desde los que tienen aplicaciones tangibles hasta los más teóricos y abstractos. Es peligroso abandonar toda esa “otra” ciencia por el mero hecho de que venda menos. Modificado con fines pedagógicos. Moral, D. (2017). Ciencia hay más de una. Recuperado el 18 de abril del 2017 en http://bit.ly/2oIPdxY Opciones: 1 Reportajecientífico 2 Noticia 3 Artículo deopinión 4 Crónica Correcta la opción:3 Artículo de opinión pregunta:72 Identifique la idea principal en el fragmento. Los apodos y sobrenombres existen porque todos los seres humanos nos prestamos para ser mencionados de tal o cual manera, porque algo, aunque sea pequeñito, nos hace únicos y diferentes, y porque estamos expuestos a la lupa de un sinnúmero de observadores. Uno de los mejores caldos de cultivo para los alias es el colegio, pero no es el único, no podemos dejar de lado a los personajes de la política, periodistas, vecinos, deportistas y familiares que seguramente serán motivo de inspiración para un nuevo artículo. León, R. (2008). De apodos, motes y sobrenombres. Revista Mundo Diners. Nro. 318. Quito. p. 93. Opciones: 1 En cuestión de apodos no podemos dejar de lado a políticos, periodistas, vecinos, deportistas y familiares que inspirarán nuevos alias 2 Todos los seres humanos, sin excepción, tenemos sobrenombres o apodos 3 Los sobrenombres existen porque somos únicos y estamos expuestos a un sinnúmero de observadores 4 Uno de los mejores caldos de cultivo para los sobrenombres es el colegio Correcta la opción:3 Los sobrenombres existen porque somos únicos y estamos expuestos a un sinnúmero de observadores pregunta:73 Identifique la idea principal del texto. Para un hispanohablante, las lenguas del mismo origen, es decir las romances o neolatinas (portugués, francés, italiano, catalán, gallego, rumano, entre otras.), al compartir muchas estructuras y léxico, resultarían más fáciles para el aprendizaje. Entonces, si la enseñanza se basa en la comparación o contraste gramatical, puede ser sencillo para una persona adulta que tenga un conocimiento básico de su propia gramática,
  • 33. aprender una lengua romance. A la hora de estudiar, a un hispanohablante puede resultarle sencilla la comprensión de otra lengua de la misma familia, como el portugués, por ejemplo, pero, según los especialistas, lo que parece fácil no suele ser siempre así. La facilidad, sobre todo para la comprensión que se presenta en un primer momento, hace que no se tengan en cuenta las dificultades porque pasan desapercibidas. Modificado con fines pedagógicos. Recuperado el 10 de febrero de 2017 en http://bbc.in/2k9ooku Opciones: 1 Aprender otra lengua de la misma familia puede resultar sencillo, pero según algunos especialistas, lo que parece fácil no suele ser siempre así 2 La facilidad para la comprensión que se presenta en un primer momento hace que las dificultades pasen desapercibidas 3 El aprendizaje de lenguas romances o neolatinas resultaría de mayor facilidad para los hablantes nativos de español 4 Si la enseñanza se basa en el contraste gramatical, puede ser sencillo para un adulto con conocimiento básico de su gramática aprender una lengua romance Correcta la opción:3 El aprendizaje de lenguas romances o neolatinas resultaría de mayor facilidad para los hablantes nativos de español pregunta:74 Identifique la idea principal del texto. Tal como no saber leer y escribir determinaba el futuro profesional y social de una persona a principios del siglo XX, hoy no manejar más que el idioma materno es una barrera insoslayable de entrada al mercado laboral para cualquier joven. Por razones económicas, un segundo idioma se ha vuelto mucho más necesario, como una herramienta de trabajo. De esta manera comenzó un desafío influenciado por la comunidad hispana de EE.UU.: integrar el español a las aulas en los distritos de Davis y Granite. No como un segundo idioma, sino en un plan que contemplara 50 % del tiempo aprendiendo las materias curriculares en inglés y 50 % en español. El programa resultó un éxito y pronto se extendió. Se lanzó a nivel estatal en 2009, cuyo año académico contó con 1 400 estudiantes. En cinco años, el programa se había ampliado a 25 000 solo en Utah, y otros estados como Delaware e Indiana comenzarán a implementarlo. Modificado con fines pedagógicos. Hola, C. (2016) Recuperado el 10 de febrero de 2017 en http://bbc.in/1mIGdEC Opciones: 1 El desconocimiento de una segunda lengua representa una barrera para los jóvenes que buscan entrar al mundo laboral 2 A inicios del siglo pasado, el dominio de la lectura y la escritura determinaba el futuro profesional y social de una persona 3
  • 34. El programa bilingüe resultó un éxito, de manera que otros estados como Delaware e Indiana comenzarán a implementarlo 4 Se integró el español a las aulas, no como un segundo idioma, sino en un plan que contemplara 50 % de las materias curriculares Correctalaopción:1 El desconocimiento de una segunda lengua representa una barrera para los jóvenes que buscan entrar al mundo laboral pregunta:75 Identifique la idea principal del texto. Puede que las personas que abandonaron su país de origen cuando eran muy pequeñas ya no recuerden su lengua natal. Sin embargo, los patrones neuronales creados por el idioma que escucharon en sus primeros años de vida permanecen intactos en su cerebro. Estos patrones se mantienen en el tiempo, incluso si la persona no ha vuelto a estar en contacto con su primera lengua. Esta huella, dejada por la lengua olvidada, podría facilitarle a quienes vivieron esta situación, el aprendizaje de su idioma natal en el futuro. En las primeras etapas del desarrollo de la lengua, los niños aprenden a distinguir -independientemente de qué lengua se trate- qué sonidos son importantes y significativos. Esta experiencia deja una suerte de representación en el cerebro, que los niños utilizan para construir su lengua nativa. Modificado con fines pedagógicos. Recuperado el 10 de febrero de 2017 en http://bbc.in/2l1b5Db Opciones: 1 La huella dejada por los patrones de la lengua materna facilita el aprendizaje futuro de esta para quienes la han olvidado 2 Los patrones neuronales creados por la lengua materna jamás se borran del cerebro humano 3 En las primeras etapas del desarrollo de la lengua, los niños aprenden a distinguir los sonidos importantes y significativos 4 Puede que los niños pequeños que abandonan su país natal no recuerden su lengua materna Correcta la opción:2 Los patrones neuronales creados por la lengua materna jamás se borran del cerebro humano pregunta:76 Con base en el texto, identifique la inferencia correcta. Silencio atronador, muerto viviente, dulce amargura, noche blanca o monstruo hermoso, son ejemplos de oxímoron, una combinación de dos palabras de significado opuesto que al unirse originan un nuevo sentido. Un estudio español publicado en la revista NeuroImage, revela que estas figuras literarias generan una intensa actividad en el área frontal izquierda del cerebro. Según los autores del estudio, "se activa la parte frontal del cerebro y se emplean más recursos de lo habitual en procesar a nivel cerebral estas expresiones". Para los experimentos, Molinaro y sus colegas crearon varias listas de frases incorrectas, neutras, oxímoron y pleonasmos. Concretamente, los investigadores han utilizado
  • 35. "monstruo geográfico" como expresión incorrecta, "monstruo solitario" como expresión neutra, "monstruo hermoso" como oxímoron, y "monstruo horrible" como pleonasmo. Los resultados muestran que cuanto menos natural es la expresión, más recursos requiere para ser procesada en la parte frontal izquierda del cerebro. Modificado con fines pedagógicos. Sanz, E. (s.f.). Recuperado el 17 de junio de 2017 en http://bit.ly/1phnMkO Opciones: 1 La actividad cerebral se incrementa cuanto menos natural es la expresión, porque requiere más recursos para ser procesada 2 El oxímoron es la única figura que genera una intensa actividad en el área frontal izquierda del cerebro 3 La parte frontal izquierda del cerebro se activa en procesos de abstracción porque debe construir un sentido inexistente 4 El oxímoron es una combinación de dos palabras de significado opuesto, que al unirse originan un nuevo sentido Correcta la opción:3 La parte frontal izquierda del cerebro se activa en procesos de abstracción porque debe construir un sentido inexistente pregunta:77 Con base en el texto, identifique la inferencia adecuada. Hace miles de años, los habitantes de los Andes vienen "domesticando" naturalmente una papa amarga que de otra manera no sería comestible y que puede conservarse durante décadas. El chuño se prepara en junio y julio, en pleno invierno del hemisferio sur, cuando llega "el friaje" al altiplano de los Andes y las temperaturas bajan en la noche hasta los -5 grados. Las comunidades andinas aprovechan el contraste de las temperaturas durante el día y la noche para "momificar" las papas. En un proceso de liofilización natural, se congelan las papas por las noches y se deshidratan al sol durante el día, a temperaturas que alcanzan los 18 grados. Una vez cosechadas las papas, normalmente en el mes de mayo, se llevan hasta las partes planas de la cordillera, llamadas chuñochinapampa. Esta elaboración artesanal, de unos 20 días de duración (incluyendo el secado), sirve para eliminar todos los alcaloides que tiene la papa y que le dan el sabor amargo original. Modificado con fines pedagógicos. Recuperado el 22 de junio de 2017 en http://bbc.in/2slifG5 Opciones: 1 Habitantes de los Andes han domesticado naturalmente una papa amarga que puede conservarse durante décadas 2 Ciertas técnicas ancestrales pueden procesar y conservar alimentos sin usar sustancias químicas o procesos industriales 3 El proceso artesanal para preparar el chuño (congelación y secado) elimina los alcaloides que dan sabor amargo a la papa 4 La única propiedad que aporta el proceso ancestral de liofilización de papas amargas es hacerlas comestibles
  • 36. Correcta la opción:2 Ciertas técnicas ancestrales pueden procesar y conservar alimentos sin usar sustancias químicas o procesos industriales pregunta:78 Con base en el texto, identifique la inferencia adecuada. Los modistos, costureras y sastres que cosen a mano tienen una agudeza visual superior a la de otras personas. La visión en 3D o estereoscópica de quienes se dedican a la costura es tan afilada como sus propias agujas. Es la capacidad que tiene nuestro cerebro para recibir la información en 2D que perciben nuestros ojos y transformarla en una imagen con profundidad o en tres dimensiones. Los profesionales de la costura tienen una precisión 80 % mayor que las personas de otras profesiones cuando calculan la distancia entre ellos mismos y los objetos que observan. No está claro si esta actividad afina la visión estereoscópica de las personas o si los sastres se ven atraídos hacia esta profesión debido a su extraordinaria agudeza visual. Ambas posibilidades, la autoselección y la experiencia, no son excluyentes, una buena estereovisión es una funcionalidad importante para ser un buen modisto, a la vez que el entrenamiento regular en tareas que requieren de una buena estereovisión mejora la agudeza visual. Modificado con fines pedagógicos. Recuperado el 22 de junio de 2017 en http://bbc.in/2tdmvHZ Opciones: 1 No está claro si la costura afina la visión estereoscópica o si los sastres eligen su profesión por su eficaz visión 2 Los modistos, las costureras y sastres poseen una visión estereoscópica tan afilada como sus propias agujas 3 El uso de máquinas de coser impide que quienes se dedican a la costura desarrollen la visión estereoscópica 4 Coser a mano podría ser una terapia eficaz para mejorar la agudeza visual de personas con problemas de estereovisión Correcta la opción:4 Coser a mano podría ser una terapia eficaz para mejorar la agudeza visual de personas con problemas de estereovisión pregunta:79 Identifique el género al que pertenece el fragmento. ¡Pues bien!, yo necesito decirte que te adoro, decirte que te quiero con todo el corazón; que es mucho lo que sufro, que es mucho lo que lloro, que ya no puedo tanto, y al grito en que te imploro, te imploro y te hablo en nombre de mi última ilusión. Acuña, M. Opciones: 1 Épico 2
  • 37. Narrativo 3 Lírico 4 Dramático Correcta la opción:3 Lírico pregunta:80 Identifique el género al que pertenece el fragmento. En 1823, antes de la partida de las jornadas de Junín y Ayacucho, inconclusa todavía la independencia política, Andrés Bello proclamaba la independencia espiritual: la primera de sus Silvas americanas es una alocución a la poesía, “maestra de los pueblos y los reyes”, para que abandone a Europa -luz y miseria- y busque en esta orilla del Atlántico el aire salubre de que gusta su nativa rustiquez. La forma es clásica; la intención es revolucionaria. Con la alocución, simbólicamente, iba a encabezar Juan María Gutiérrez nuestra primera grande antología, la América poética de 1846. Opciones: 1 Narrativo 2 Ensayístico 3 Dramático 4 Lírico Correcta la opción:2 Ensayístico pregunta:81 Identifique el género literario al que corresponde el texto. Volverán las oscuras golondrinas en tu balcón sus nidos a colgar, y, otra vez, con el ala a sus cristales jugando llamarán; pero aquellas que el vuelo refrenaban tu hermosura y mi dicha al contemplar, aquellas que aprendieron nuestros nombres... ésas... ¡novolverán! Opciones: 1 Lírico 2 Narrativo 3 Dramático 4 Épico
  • 38. Correcta la opción:1 Lírico pregunta:82 ¿A qué género literario pertenece el texto? Entretanto Neptuno advirtió por el ruido tan grande que el mar se agitaba, se desataba la tormenta y el agua volvía de los profundos abismos y, gravemente afectado, miró desde lo alto sacando su plácida cabeza por encima del agua. Ve por todo el mar la flota deshecha de Eneas, y a los troyanos atrapados por las olas y la ruina del cielo; y no se le escaparon al hermano las trampas y la ira de Juno. Así que llama ante él al Céfiro y al Euro, y así les dice: «¿A tanto ha llegado el orgullo de la raza vuestra? ¿Ya revolvéis el cielo y la tierra sin mi numen, vientos, y os atrevéis a levantar moles tangrandes? Opciones: 1 Épico 2 Lírico 3 Cómico 4 Cuento Correcta la opción:1 Épico pregunta:83 Identifique el subgénero al que pertenece el fragmento. HAMLET: ¡Ah, ya muero, Horacio! El fuerte veneno señorea mi ánimo. No viviré para oír las nuevas de Inglaterra, pero adivino que será elegido rey Fortinbrás. Le doy mi votoagonizante. Díselo, junto con todos los sucesos que me han llevado... El resto es silencio. [Lanza un hondo suspiro y ] muere. HORACIO: Ha estallado un noble pecho. Buenas noches, buen príncipe; que cánticos de ángeles te lleven al reposo. ¿Por qué vienen los tambores? Opciones: 1
  • 39. Comedia 2 Drama 3 Tragicomedia 4 Tragedia Correcta la opción:4 Tragedia pregunta:84 Identifique el subgénero literario al que pertenece el fragmento No hay extensión más grande que mi herida, lloro mi desventura y sus conjuntos y siento más tu muerte que mi vida. Ando sobre rastrojos de difuntos, y sin calor de nadie y sin consuelo voy de mi corazón a mis asuntos. Temprano levantó la muerte el vuelo, temprano madrugó la madrugada, temprano estás rodando por el suelo. Hernández,M.(s.f.) Opciones: 1 Oda 2 Égloga 3 Elegía 4 Romance Correcta la opción:3 Elegía pregunta:85 Identifique el subgénero al que pertenece el fragmento. Un pequeño grupo, entre el que está don Américo, acompaña paso a paso el ataúd donde están los restos de quien en vida fue don Calvo Figueroa, carpintero de profesión, guitarrista por vocación, viudo de la Mercedes, padre de cuatro hijos varones y seis mujeres, abuelo de nadie sabe cuántos nietos, persona excelente y uno de los grandes del pueblo, pues medía un metro con 89 centímetros. Don Américo, en un momento de silencio total en el que hasta los pajaritos dejaron de cantar, dice con voz pausada, contundente y sonora:
  • 40. - Pensar que ayer mi compadre estaba lleno de vida… y ahora está perfectamente muerto… pensar que lo único perfecto que hacemos en la vida es morirnos; eso sí, los mejores, como mi compadre, mueren después de haber sido buenos, amables y generosos. Andrade Heymann, J. (2010). Cabizmundo y meditabajo. Pág. 270. Opciones: 1 Fábula 2 Cuento 3 Epopeya 4 Épica Correcta la opción:2 Cuento pregunta:86 Seleccione las características del realismo literario. 1. El hombre como centro de cultura y saber 2. Descripción objetiva de la vida ordinaria 3. Búsqueda de la belleza de las cosas 4. Enjuiciamiento crítico de la sociedad 5. Presentación de personajes de distintas clases sociales 6. Conocimiento de autores griegos y latinos Opciones: 1 1, 2, 4 2 1, 3, 6 3 2, 4, 5 4 3, 5, 6 Correctalaopción:3 2, 4, 5 pregunta:87 Seleccione las características determinantes de una obra simbolista. 1. Emplea de forma recurrente la figura de la imagen 2. Se describe de forma minuciosa la realidad 3. Se elude nombrar directamente los objetos 4. Emplea de forma recurrente la figura del hipérbaton Opciones:
  • 41. 1 1, 3 2 1, 4 3 2, 3 4 2, 4 Correctalaopción:1 1, 3 pregunta:88 Relacione la escuela literaria con su característica. Escuela Característica 1. Romanticismo a) La libertad de pensamiento y la idealización de la naturaleza fueron las fuentes de inspiración 2. Clasicismo b) Exalta el individualismo. Hay cambios en los temas y sus formas. Se caracterizó por retomar los valores de la cultura grecorromana 3. Renacimiento c) Las composiciones se caracterizaron por ser equilibradas, sobrias y armónicas Opciones: 1 1a, 2c, 3b 2 1b, 2a, 3c 3 1b, 2c, 3a 4 1c, 2a, 3b Correcta la opción:1 1a, 2c, 3b pregunta:89 Identifique la escuela literaria que se destaca por su intención renovadora, de avance y exploración. Opciones: 1 Clasicismo 2 Romanticismo 3 Vanguardismo 4 Realismo Correcta la opción:3 Vanguardismo
  • 42. pregunta:90 La escuela literaria del modernismo se caracteriza por: Opciones: 1 exceder los límites aceptados como norma cultural europea de inicios del siglo XX 2 priorizar los sentimientos del individuo como respuesta al racionalismo y clasicismo europeo 3 surgir en América Latina a finales del siglo XIX bajo influencia del parnasianismo y simbolismo 4 reproducir la realidad de forma casi documental en todos los aspectos, tanto sublimes como vulgares Correcta la opción:3 surgir en América Latina a finales del siglo XIX bajo influencia del parnasianismo y simbolismo pregunta:91 Identifique al autor del fragmento. Revestir sabe el vino los más sórdidos antros De un milagroso lujo, Y hace surgir más de un pórtico fabuloso Entre el oro de su rojo vapor, Como el sol que se pone en un cielo nublado Agranda el opio aquello que no tolera límites, Lo ilimitado alarga, El tiempo profundiza, los deleites ahonda, Y de placer triste y oscuro, Anega y colma al alma rebasada. Mas todo eso no vale el prodigio terrible De tus ojos, de tus verdes ojos, Lagos donde mi alma tiembla y se ve invertida… Llegan mis sueños en tropel Para abrevar en esos dos abismos amargos Mas todo esto no vale prodigio terrible De tu mordiente saliva, Que sume en el olvido a mi alma impenitente Y, el vértigo arrastrando, La trae desfallecida a orillas de la muerte. Opciones: 1 Rubén Darío 2 Paul Verlaine 3 Fernando Pessoa
  • 43. 4 Charles Baudelaire Correcta la opción:4 Charles Baudelaire pregunta:92 Seleccione las obras adscritas al clasicismo literario. 1. Soledades 2. LasGeórgicas 3. Las Bucólicas 4. Los viajes de Gulliver Opciones: 1 1, 3 2 1, 4 3 2, 3 4 2, 4 Correctalaopción:3 2, 3 pregunta:93 Elija las obras adscritas al barroco literario. 1. La vida del Buscón 2. Robinson Crusoe 3. Fábula de Polifemo y Galatea 4. Aventuras del Capitán Singleton Opciones: 1 1, 3 2 1, 4 3 2, 3 4 2, 4 Correctalaopción:1 1, 3 pregunta:94 Relacione la escuela literaria con su obra representativa. Escuela Obra
  • 44. Opciones: 1 1a, 2b, 3c, 4d 2 1b, 2a, 3d, 4c 3 1c, 2d, 3a, 4b 4 1d, 2c, 3b, 4a 1. Culteranismo a) Volverán las oscuras golondrinas 2. Romanticismo b) Polifemo y Galatea 3. Realismo c) La tía Tula 4. Modernismo d) Pepita Jiménez Correcta la opción:2 1b, 2a, 3d, 4c pregunta:95 Relacione el autor con la corriente literaria a la que pertenece. Autor Corriente Opciones: 1 1a, 2d, 3b, 4c 2 1b, 2a, 3c, 4d 3 1c, 2b, 3d, 4a 4 1d, 2c, 3a, 4b Correcta la opción:4 1d, 2c, 3a, 4b pregunta:96 ¿A qué corriente literaria pertenece el fragmento? En lo físico, Grandet era hombre de cinco pies, rechoncho, cuadrado, con unas pantorrillas de doce pulgadas de circunferencia, grandes rótulas y anchas espaldas; su cara era redonda, curtida y marcada por la viruela; su barba era recta, sus labios no ofrecían ninguna sinuosidad y sus dientes eran blancos; sus ojos tenían la expresión tranquila y devoradora que el pueblo atribuye al basilisco; su frente, llena de arrugas transversales, no carecía de significativas protuberancias; y sus cabellos, rubios y blancos, eran de color plata y oro, al decir de algunas gentes que no conocían la gravedad que podía tener el hecho de gastar una broma al señor 1. Federico García Lorca 2. Tristan Tzara 3. Vicente HuidobroFernández 4. Guillermo de Torre a) Creacionismo b) Ultraísmo c) Dadaísmo d) Surrealismo
  • 45. Grandet. Su nariz, gorda por la punta, sostenía un lobanillo veteado que, según decía el vulgo, y no sin razón, estaba lleno de malicia. Esta cara anunciaba esa astucia peligrosa, esa fría probidad y ese egoísmo del hombre acostumbrado a concentrar sus sentimientos en el único ser que le fue siempre querido, en su hija Eugenia, en su única heredera. Por otra parte, la actitud, los modales, el paso, todo en él confirmaba esa creencia en sí que da la costumbre de ver que se sale siempre airoso en sus empresas; así, pues, aunque el señor Grandet era, en apariencia, hombre de costumbres sencillas y afeminadas, tenía un carácter dehierro. Balzac. (1833) Eugenia Grandet. Editorial No Books. Opciones: 1 Simbolismo 2 Naturalismo 3 Realismo 4 Romanticismo Correcta la opción:3 Realismo pregunta:97 Identifique la corriente literaria a la que pertenece el fragmento. El albatros (fragmento) Este alado viajero, ¡qué inútil y qué débil! Él, otrora tan bello, ¡qué feo y qué grotesco! ¡Este quema su pico, sádico, con la pipa, Aquél, mima cojeando al planeador inválido! El poeta es igual a este señor del nublo, Que habita la tormenta y ríe del ballestero. Exiliado en la tierra, sufriendo el griterío, Sus alas de gigante le impiden caminar. Baudelaire, C. (1851) El Albatros. Opciones: 1 Barroco 2 Simbolismo 3 Culteranismo 4 Conceptismo Correcta la opción:2 Simbolismo
  • 46. pregunta:98 Relacione el autor con la corriente literaria correspondiente. Autor Corriente 1. André Breton a) Expresionismo Opciones: 1 1a, 2b, 3c 2 1a, 2c, 3b 3 1b, 2a, 3c 4 1c, 2a, 3b 2. Franz Kafka b) Dadaísmo 3. Hugo Ball c) Surrealismo Correcta la opción:4 1c, 2a, 3b pregunta:99 Con base en el fragmento, identifique la corriente a la que pertenece el poema. Vehemente dios de una raza de acero, automóvil ebrio de espacio, que piafas y te estremeces de angustia tascando el freno con estridentes dientes… Formidable monstruo japonés de ojos de fragua, nutrido de llama y de aceites minerales, ávido de horizontes y presas siderales… ¡yo desencadeno tu corazón que golpea diabólicamente, desencadeno tus gigantescos neumáticos, para la danza que sabes danzar en los blancos caminos de todo el mundo! Marinetti, F. T. El automóvil de carrera. Recuperado el 28 de agosto de 2014 en http://bit.ly/YpoIxj Opciones: 1 Surrealismo 2 Futurismo 3 Expresionismo 4
  • 47. Dadaísmo Correcta la opción:2 Futurismo pregunta:100 Elija los poemas producidos durante el periodo modernista ecuatoriano. 1. El alma en los labios 2. A unos ojos hermosos 3. Primavera mística y lunar 4. A Carmen 5. Canto a Bolívar 6. Romanza de las horas Opciones: 1 1, 2, 5 2 1, 3, 6 3 2, 4, 5 4 3, 4, 6 Correctalaopción:2 1, 3, 6 pregunta:101 Relacione la autora con su obra. Opciones: 1 1a, 2c, 3b 2 1b, 2c, 3a 3 1c, 2a, 3b 4 1c, 2b, 3a Autora Obra 1. Alicia Yánez Cossío a) Así se hace una mamá 2. Teresa León de Noboa b) Los gatos literatos 3. Catalina Sojos c) Bruna, soroche y los tíos Correcta la opción:4 1c, 2b, 3a pregunta:102 Relacione el autor con su obra.
  • 48. Opciones: 1 1a, 2b, 3c 2 1a, 2c, 3b 3 1b, 2c, 3a 4 1c, 2b, 3a Autor Obra 1. Humberto Salvador a) Entre Marx y una mujer desnuda 2. Jorge Velasco Mackenzie b) El rincón de los justos 3. Jorge Enrique Adoum c) En la ciudad he perdido una novela Correcta la opción:4 1c, 2b, 3a pregunta:103 Relacione la obra literaria ecuatoriana con su autor. Opciones: 1 1a, 2c, 3b, 4d 2 1b, 2a, 3d, 4c 3 1d, 2a, 3b, 4c 4 1d, 2c, 3b, 4a Correcta la opción:4 1d, 2c, 3b, 4a pregunta:104 Elija las obras del escritor Albert Camus. 1. El proceso 2. La náusea 3. La peste 4. El extranjero Opciones: Obra Autor 1. Los Sangurimas a) EliécerCárdenas 2. La emancipada b) Luis A. Martínez 3. A la Costa c) Miguel Riofrío 4. Polvo y ceniza d) José de la Cuadra
  • 49. 1 1, 2 2 1, 3 3 2, 4 4 3, 4 Correctalaopción:4 3, 4 pregunta:105 Elija las obras del autor John Ronald Reuel Tolkien. 1. Harry Potter 2. El Señor de los Anillos 3. Las Crónicas de Narnia 4. El Hobbit Opciones: 1 1, 3 2 1, 4 3 2, 3 4 2, 4 Correctalaopción:4 2, 4 pregunta:106 Identifique el autor al que hace referencia el texto. La mayoría de sus narraciones se desarrollan en San Martín, una ciudad ficticia al borde de un río. Entre sus obras emblemáticas se encuentran: La vida breve (1950), donde Brausen, el protagonista, enclaustrado en su apartamento, construye una ficción que le permitirá ser otro; Juntacadáveres (1964), que relata un fragmento de la vida de Larsen cuando abre un burdel en San Martín; El astillero (1961) que se considera su obra maestra, donde Larsen se convierte en un ejecutivo muy bien remunerado; mientras que Dejemos hablar al viento (1979) representa el fracaso del hombre. Modificado con fines pedagógicos. Becerra, E. (s/f). Historia del otro lado. Recuperado el 2 de abril de 2017 en https://goo.gl/agxd3a Opciones: 1 José Donoso 2 Octavio Paz 3
  • 50. Juan Onetti 4 Carlos Fuentes Correcta la opción:3 Juan Onetti pregunta:107 Identifique la clase de rima que hay en la estrofa. Bajo el dosel de gigantesca roca yace el titán, cual Cristo en el Calvario, marmóreo, indiferente y solitario, sin que brote el gemido de su boca. Del Casal, J. Prometeo. Opciones: 1 Continua 2 Cruzada 3 Abrazada 4 Pareada Correcta la opción:3 Abrazada pregunta:108 Identifique la clase de rima que se aplica en la estrofa. Dolores, costurera de mi casa añosa de mi casa, vieja amiga; era tu corazón crujiente miga de pan; eran tus ojos lenta brasa. Panero, L. (s.f.). Recuperado el 13 de agosto de 2013 en http://roble.pntic.mec.es/msanto1/lengua/estrofas.htm Opciones: 1 Asonante 2 Consonante 3 Ritmo 4 Libre Correcta la opción:2
  • 51. Consonante pregunta:109 Identifique la clase de rima. Fue en un poniente mágico de púrpura y oros: con música de brisas en los pinos sonoros, rítmicas desfilaban las horas, al ocaso, tal una ronda griega cincelada en un vaso Silva, M. (s.f.). La investidura. Recuperado el 29 de marzo de 2017 en http://bit.ly/2nfxoT5 Opciones: 1 Pareada 2 Continua 3 Abrazada 4 Encadenada Correcta la opción:1 Pareada pregunta:110 Identifique la clase de rima. Largo espectro de plata conmovida el viento de la noche suspirando, abrió con mano gris mi vieja herida y se alejó: yo estaba deseando. Llaga de amor que me dará la vida perpetua sangre y pura luz brotando. García, F. (s.f.). Largo espectro de plata conmovida. Recuperado el 29 de marzo de 2017 en http://bit.ly/2nfAvu9 Opciones: 1 Encadenada 2 Pareada 3 Continua 4 Abrazada Correcta la opción:1 Encadenada pregunta:111 Identifique la clase de rima.
  • 52. Hombres necios que acusáis a la mujer sin razón, sin ver que sois la ocasión de lo mismo que culpáis De la Cruz, I. (s.f.). Redondillas. Recuperado el 29 de marzo de 2017 en http://bit.ly/2mOva1I Opciones: 1 Pareada 2 Continua 3 Encadenada 4 Abrazada Correcta la opción:4 Abrazada pregunta:112 Identifique la métrica del verso en negrita. Un mozo trae por un sendero sus herramientas y su morral: otro con caites y sin sombrero busca una vaca con su ternero para ordeñarla junto al corral. Rubén Darío. Del trópico. Recuperado el 10 de febrero de 2016 en http://bit.ly/1O3YKRL Opciones: 1 Decasílabo 2 Octosílabo 3 Endecasílabo 4 Dodecasílabo Correcta la opción:1 Decasílabo pregunta:113 ¿Qué tipo de versos tiene esta estrofa? Poesía XLIV
  • 53. Tiene el leopardo un abrigo en su monte seco y pardo yo tengo más que el leopardo porque tengo un buen amigo Martí, J. (1891). Versos sencillos. Recuperado el 13 de agosto de 2013 en http://www.damisela.com/literatura/pais/cuba/autores/marti/sencillo/xliv.htm Opciones: 1 Eneasílabos 2 Octosílabos 3 Endecasílabos 4 Hexasílabos Correcta la opción:2 Octosílabos pregunta:114 Identifique el verso octosílabo de la autora Concha Benito. Opciones: 1 Dime tú, ahora 2 De tu cabeza de águila afiebrada 3 Del fauno ingrato que un día 4 De tu seso enjambrado Correcta la opción:3 Del fauno ingrato que un día pregunta:115 Con base en el fragmento, identifique la métrica de la estrofa. ¡Oh dulces prendas, por mi mal halladas, dulces y alegres cuando Dios quería! Juntas estáis en la memoria mía, y con ella en mi muerte conjuradas. De la Vega, G. Soneto X. Recuperado el 28 de agosto de 2014 en http://bit.ly/1C7zdVo Opciones: 1 Endecasílabo 2 Decasílabo
  • 54. 3 Eneasílabo 4 Dodecasílabo Correcta la opción:1 Endecasílabo pregunta:116 Identifique el tipo de métrica presente en la estrofa. Era una tarde de enero; el sol casi se ocultaba, y las brisas dulcemente gemían entre las ramas. Darío, R. Romance. Recuperado el 15 de agosto de 2014 en http://bit.ly/1oYxIom Opciones: 1 Heptasílabo 2 Decasílabo 3 Octosílabo 4 Eneasílabo Correcta la opción:3 Octosílabo pregunta:117 Relacione el periodo de la literatura ecuatoriana con su obra representativa. Opciones: 1 1a, 2d, 3c, 4b 2 1b, 2a, 3c, 4d 3 1d, 2a, 3b, 4c 4 1d, 2b, 3a, 4c Periodo Obra 1. Precolonial a) La conquista de Menorca 2. Colonial b) Oda a Miñarica 3. Independentista c) A mis enemigos 4. Republicano d) Atahualpa Huañui
  • 55. Correcta la opción:3 1d, 2a, 3b, 4c pregunta:118 ¿En qué verso se usa el polisíndeton? Opciones: 1 Y el odio y el furor de los tiranos 2 En tierra, en humo, en polvo, en sombra, en nada 3 El huracán aullaba como mastín de caza 4 El Chimborazo alzaba su cabeza de abuelo Correcta la opción:1 Y el odio y el furor de los tiranos pregunta:119 Elija los versos que contienen retruécano. 1. Cortante como una daga 2. Voy a prenderme fuego sin llamas, ahora que no me llamas ni para pedirme fuego 3. Ni dice lo que siente ni siente lo que dice 4. Millones de ideas lo atacaban 5. No rezo para vivir ni vivo para rezar 6. Es tan corto el amor y tan largo el olvido Opciones: 1 1, 3, 4 2 1, 4, 6 3 2, 3, 5 4 2, 5, 6 Correctalaopción:3 2, 3, 5 pregunta:120 Con base en el fragmento, identifique la figura literaria presente en la estrofa. Tum tum, ¿quién es? una rosa y un clavel abre la muralla. Tum tum, ¿quién es? el sable del coronel cierra la muralla.
  • 56. Guillén, N. La muralla. Recuperado el 28 de agosto de 2014 en http://bit.ly/VOzuME Opciones: 1 Asindetón 2 Onomatopeya 3 Elipsis 4 Síncopa Correcta la opción:2 Onomatopeya